Best strategy against anti-material monsters











up vote
14
down vote

favorite
4












For my story the inhabitants of the setting find themselves fighting against endless hordes of monsters. But these monsters have several traits that will require unique strategies against them which I am trying to work out.



The setting
First of all some background on the world they're living in. Its set in a large kingdom about half the size of western Europe and is surrounded by barrens. These barrens are infested with the monsters and are constantly attacking. The area in between the kingdom and barrens is known as the frontier. A massive stretch of land filled with walls, fortresses and other defenses. If one of them were to fall it would be a simple matter to fall back. The tech level is undecided. I was thinking late second industrial revolution but medieval is also a possibility.



Magic
The setting does have a magic system but it is not too relevant to the tactics. It is of a runic nature and by drawing on ley lines anythign within the parameters of the rune can be done. It is not that suitable for use by units and is usually reserved for reinforcing defenses or enhancing long range weaponry like artillery.



The Monsters
The monsters are the creatures of a God of the void. They desire to wipe out everything and they are created by this god for doing so. They have multiple traits that make them difficult to stop.




  • Attraction to magic: The world is held together by magic so they have an ability to seek it out and destroy it. Magic can be found in all living things, running through the landscape as ley lines and in runic circles made by mages. This draws them endlessly into the kingdom which still has functional magic thanks to its defense from the monsters.

  • Anti-material: Anything the monsters touch crumbles into dust/ash. This severely limits the effectiveness of different weapons. Melee weapons will wear out quickly in combat and armour will crumble after a single hit. This obviously makes ranged weapons a smarter option. Defenses like walls can have some effectiveness at holding them back depending on thickness. They will need to be constantly patched and repaired however with constant attacks. Due to the magic running within living creatures their crumbling effect is different on them. Organics will wither instead. Organs will stop and limbs will become useless. Damage depends on amount of exposure, limited exposure will leave the tissue functional if scarred and crippled. Large exposure will leave tissue dead and after that it will crumble just like anything else.

  • Endless variety/numbers: There is no end in sight to the numbers of the monsters, society has basically just been holding them at bay at the frontier. They can come in many different shapes and sizes, appearing both like real animals and mythical ones.

  • No organs: Due to their nature as monsters of the void, they do not require anything to survive and therefore have no vital organs. Killing them is dependent on body damage. Cutting them to pieces and blowing them up are effective at killing them. Ranged weapons like guns generally don't do much more than slow them down. Although higher caliber and higher spread guns are effective. Large amounts of small wounds have also been shown to kill them, as if they were bleeding out, although no blood has ever been observed from them.




I need to work out what tactics would be best against these monsters. Both on an organizational army wide scale and independent unit scale ie what training and equipment soldiers receive and what tactics the army uses.



EDIT:



Alright I'll give you some extra details and clear some things up.




  • Withering organics is not an extra thing that happens before crumbling to ash. Its just the same thing happening in a different way.

  • touching a single part of an object will not cause the entire thing to crumble, only the touched area. Parts surrounding it will weaken however, monsters must use repeated attacks and touches to destroy things. This means the size of an object matters, a bullets small size will lead to near immediate disintegration upon entering their body but a thick stone wall could last a siege.

  • Their disintegration ability is in effect over the entire body, touch it anywhere and you will crumble. They are immune to the effect themselves so no friendly fire.

  • As creatures of the void, they are not all physically there. Hence no need for organs and such. While they can be touched and killed, they do not fit in with the rest of nature. Basically eldritch monsters. They also appear to fade away into smoke upon death, leaving no bodies.

  • The durability of the object matters when crumbling. e.g. stone lasts better than clay. Metal weapons can survive a few hits(as long as the structure is still sound)

  • There are monsters capable of things like flight, swimming and burrowing through the ground. Their numbers vary. Tunneling monsters are uncommon but do exist, they are more common in areas with plenty to dig through like mountains. It takes a while for them to dig but still faster than any natural earth creature thanks to their disintegration. Flyers are common but still less than standard land creatures. Anti-air weaponry would be effective against them.
    Oceans and water bodies in monster territory tend to be teeming with them. Sailing in monster waters is effectively suicide since they can destroy the bottoms of ships. The only sailing activity is limited to a safe and secure bay which is kept clean of monsters as well as freshwater bodies deep with safe territory.

  • The lower density of liquids and gases leads to the disintegration being less useful. e.g. water will become lifeless and less buoyant while air becomes stale and thin.

  • Monsters can tone down their ability at will, this is mostly done for convenience though, not much use walking when the ground is gone.

  • The origin of the monsters is not known by society. It appears they just spawn within their own territories. Created by their god.

  • Monster strategies usually come down to either strong individuals or pack hunting. Packs will usually stick to monsters of similar body types but this is not guaranteed. Monsters can form large hordes of many types which will attack in waves. Their God does little to lead them past simply creating them.

  • Monster intelligence can vary, most are nothing more than primal creatures whose only instinct is to seek out magic and destroy. But some can rise higher to use strategy in combat or lead their fellows like an alpha. Some can even organize entire armies of monsters but these are rare and their tactics pretty simplistic outside of maneuvering.

  • A single type of monster has developed which is capable of mimicking humans and infiltrating society to bring it down from inside. Their existence is not known to humans so tactics don't really exist against them.

  • Humanities goals has basically just been survive for their entire history since the monsters first started showing up. All they have been able to do. While they have fallen back and reclaimed bits of the frontier many times, they have never really been able to advance much further. Doing something about them will be the main plot of the story but for now I want to know the tactics already in use.

  • Very frontlines forts can expect 2-5 attacks a week, they use the time between attacks to recover, repair and restock.

  • It is not uncommon for groups of monsters to make their way past the frontline, this is why its so important for extra fortifications behind them. So recorded monster has made it all the way to the kingdom through the entire frontier's defenses.

  • Society is geared towards supporting the frontier, with resources and troops being sent out as often as possible. Internal politics can occasionally decrease this supply(because politicians can be bickering idiots at times)

  • Runes can not be freestanding or created instantly. They must be drawn onto something. So good for reinforcing a wall or adding an extra effect to it, not so much for force fields. Using them in the defense also has the extra drawback of attracting more monsters

  • The monsters cover basically the entire world except for this kingdom. Their more dense around the kingdom itself since the barrens don't really have any magic to attract them. It is theorized that there may be other kingdoms out there which have managed to survive like them. But between the monsters and distance contact it not viable until the threat is dealt with.

  • Terrain will regenerate over time when magic is available. Ley lines will naturally create features of the terrain and said features will simultaneously reinforce the ley lines. E.g. rivers and ridges are more likely to end up running over ley lines. This means that within the barrens the monsters have destroyed most features of the landscape to weaken and destroy the ley lines. As long as the kingdom still stands, the ley lines of the world will continue to regenerate using them as a source while simultaneously being targeted by monsters.

  • If the monsters were successful in destroying all magic including ley lines on the planet, it would die and come apart. If every planet in a system lost its lines then its star would be turned into a black hole(systems and planets function as gigantic runes keeping the universe together). The God of the void aims to do this everywhere

  • The monsters were not always attacking. There was a period where there was a good god protecting them until he got preoccupied with keeping the universe together elsewhere. They had gotten to about bronze age tech before the monsters started appearing. Much of the world was swiftly lost and they retreated into the fortified kingdom.




Phew, the number of things you only think to add after posting, well here we go. Time to say if anyone points out more details I could add....










share|improve this question
























  • Comments are not for extended discussion; this conversation has been moved to chat.
    – James
    yesterday















up vote
14
down vote

favorite
4












For my story the inhabitants of the setting find themselves fighting against endless hordes of monsters. But these monsters have several traits that will require unique strategies against them which I am trying to work out.



The setting
First of all some background on the world they're living in. Its set in a large kingdom about half the size of western Europe and is surrounded by barrens. These barrens are infested with the monsters and are constantly attacking. The area in between the kingdom and barrens is known as the frontier. A massive stretch of land filled with walls, fortresses and other defenses. If one of them were to fall it would be a simple matter to fall back. The tech level is undecided. I was thinking late second industrial revolution but medieval is also a possibility.



Magic
The setting does have a magic system but it is not too relevant to the tactics. It is of a runic nature and by drawing on ley lines anythign within the parameters of the rune can be done. It is not that suitable for use by units and is usually reserved for reinforcing defenses or enhancing long range weaponry like artillery.



The Monsters
The monsters are the creatures of a God of the void. They desire to wipe out everything and they are created by this god for doing so. They have multiple traits that make them difficult to stop.




  • Attraction to magic: The world is held together by magic so they have an ability to seek it out and destroy it. Magic can be found in all living things, running through the landscape as ley lines and in runic circles made by mages. This draws them endlessly into the kingdom which still has functional magic thanks to its defense from the monsters.

  • Anti-material: Anything the monsters touch crumbles into dust/ash. This severely limits the effectiveness of different weapons. Melee weapons will wear out quickly in combat and armour will crumble after a single hit. This obviously makes ranged weapons a smarter option. Defenses like walls can have some effectiveness at holding them back depending on thickness. They will need to be constantly patched and repaired however with constant attacks. Due to the magic running within living creatures their crumbling effect is different on them. Organics will wither instead. Organs will stop and limbs will become useless. Damage depends on amount of exposure, limited exposure will leave the tissue functional if scarred and crippled. Large exposure will leave tissue dead and after that it will crumble just like anything else.

  • Endless variety/numbers: There is no end in sight to the numbers of the monsters, society has basically just been holding them at bay at the frontier. They can come in many different shapes and sizes, appearing both like real animals and mythical ones.

  • No organs: Due to their nature as monsters of the void, they do not require anything to survive and therefore have no vital organs. Killing them is dependent on body damage. Cutting them to pieces and blowing them up are effective at killing them. Ranged weapons like guns generally don't do much more than slow them down. Although higher caliber and higher spread guns are effective. Large amounts of small wounds have also been shown to kill them, as if they were bleeding out, although no blood has ever been observed from them.




I need to work out what tactics would be best against these monsters. Both on an organizational army wide scale and independent unit scale ie what training and equipment soldiers receive and what tactics the army uses.



EDIT:



Alright I'll give you some extra details and clear some things up.




  • Withering organics is not an extra thing that happens before crumbling to ash. Its just the same thing happening in a different way.

  • touching a single part of an object will not cause the entire thing to crumble, only the touched area. Parts surrounding it will weaken however, monsters must use repeated attacks and touches to destroy things. This means the size of an object matters, a bullets small size will lead to near immediate disintegration upon entering their body but a thick stone wall could last a siege.

  • Their disintegration ability is in effect over the entire body, touch it anywhere and you will crumble. They are immune to the effect themselves so no friendly fire.

  • As creatures of the void, they are not all physically there. Hence no need for organs and such. While they can be touched and killed, they do not fit in with the rest of nature. Basically eldritch monsters. They also appear to fade away into smoke upon death, leaving no bodies.

  • The durability of the object matters when crumbling. e.g. stone lasts better than clay. Metal weapons can survive a few hits(as long as the structure is still sound)

  • There are monsters capable of things like flight, swimming and burrowing through the ground. Their numbers vary. Tunneling monsters are uncommon but do exist, they are more common in areas with plenty to dig through like mountains. It takes a while for them to dig but still faster than any natural earth creature thanks to their disintegration. Flyers are common but still less than standard land creatures. Anti-air weaponry would be effective against them.
    Oceans and water bodies in monster territory tend to be teeming with them. Sailing in monster waters is effectively suicide since they can destroy the bottoms of ships. The only sailing activity is limited to a safe and secure bay which is kept clean of monsters as well as freshwater bodies deep with safe territory.

  • The lower density of liquids and gases leads to the disintegration being less useful. e.g. water will become lifeless and less buoyant while air becomes stale and thin.

  • Monsters can tone down their ability at will, this is mostly done for convenience though, not much use walking when the ground is gone.

  • The origin of the monsters is not known by society. It appears they just spawn within their own territories. Created by their god.

  • Monster strategies usually come down to either strong individuals or pack hunting. Packs will usually stick to monsters of similar body types but this is not guaranteed. Monsters can form large hordes of many types which will attack in waves. Their God does little to lead them past simply creating them.

  • Monster intelligence can vary, most are nothing more than primal creatures whose only instinct is to seek out magic and destroy. But some can rise higher to use strategy in combat or lead their fellows like an alpha. Some can even organize entire armies of monsters but these are rare and their tactics pretty simplistic outside of maneuvering.

  • A single type of monster has developed which is capable of mimicking humans and infiltrating society to bring it down from inside. Their existence is not known to humans so tactics don't really exist against them.

  • Humanities goals has basically just been survive for their entire history since the monsters first started showing up. All they have been able to do. While they have fallen back and reclaimed bits of the frontier many times, they have never really been able to advance much further. Doing something about them will be the main plot of the story but for now I want to know the tactics already in use.

  • Very frontlines forts can expect 2-5 attacks a week, they use the time between attacks to recover, repair and restock.

  • It is not uncommon for groups of monsters to make their way past the frontline, this is why its so important for extra fortifications behind them. So recorded monster has made it all the way to the kingdom through the entire frontier's defenses.

  • Society is geared towards supporting the frontier, with resources and troops being sent out as often as possible. Internal politics can occasionally decrease this supply(because politicians can be bickering idiots at times)

  • Runes can not be freestanding or created instantly. They must be drawn onto something. So good for reinforcing a wall or adding an extra effect to it, not so much for force fields. Using them in the defense also has the extra drawback of attracting more monsters

  • The monsters cover basically the entire world except for this kingdom. Their more dense around the kingdom itself since the barrens don't really have any magic to attract them. It is theorized that there may be other kingdoms out there which have managed to survive like them. But between the monsters and distance contact it not viable until the threat is dealt with.

  • Terrain will regenerate over time when magic is available. Ley lines will naturally create features of the terrain and said features will simultaneously reinforce the ley lines. E.g. rivers and ridges are more likely to end up running over ley lines. This means that within the barrens the monsters have destroyed most features of the landscape to weaken and destroy the ley lines. As long as the kingdom still stands, the ley lines of the world will continue to regenerate using them as a source while simultaneously being targeted by monsters.

  • If the monsters were successful in destroying all magic including ley lines on the planet, it would die and come apart. If every planet in a system lost its lines then its star would be turned into a black hole(systems and planets function as gigantic runes keeping the universe together). The God of the void aims to do this everywhere

  • The monsters were not always attacking. There was a period where there was a good god protecting them until he got preoccupied with keeping the universe together elsewhere. They had gotten to about bronze age tech before the monsters started appearing. Much of the world was swiftly lost and they retreated into the fortified kingdom.




Phew, the number of things you only think to add after posting, well here we go. Time to say if anyone points out more details I could add....










share|improve this question
























  • Comments are not for extended discussion; this conversation has been moved to chat.
    – James
    yesterday













up vote
14
down vote

favorite
4









up vote
14
down vote

favorite
4






4





For my story the inhabitants of the setting find themselves fighting against endless hordes of monsters. But these monsters have several traits that will require unique strategies against them which I am trying to work out.



The setting
First of all some background on the world they're living in. Its set in a large kingdom about half the size of western Europe and is surrounded by barrens. These barrens are infested with the monsters and are constantly attacking. The area in between the kingdom and barrens is known as the frontier. A massive stretch of land filled with walls, fortresses and other defenses. If one of them were to fall it would be a simple matter to fall back. The tech level is undecided. I was thinking late second industrial revolution but medieval is also a possibility.



Magic
The setting does have a magic system but it is not too relevant to the tactics. It is of a runic nature and by drawing on ley lines anythign within the parameters of the rune can be done. It is not that suitable for use by units and is usually reserved for reinforcing defenses or enhancing long range weaponry like artillery.



The Monsters
The monsters are the creatures of a God of the void. They desire to wipe out everything and they are created by this god for doing so. They have multiple traits that make them difficult to stop.




  • Attraction to magic: The world is held together by magic so they have an ability to seek it out and destroy it. Magic can be found in all living things, running through the landscape as ley lines and in runic circles made by mages. This draws them endlessly into the kingdom which still has functional magic thanks to its defense from the monsters.

  • Anti-material: Anything the monsters touch crumbles into dust/ash. This severely limits the effectiveness of different weapons. Melee weapons will wear out quickly in combat and armour will crumble after a single hit. This obviously makes ranged weapons a smarter option. Defenses like walls can have some effectiveness at holding them back depending on thickness. They will need to be constantly patched and repaired however with constant attacks. Due to the magic running within living creatures their crumbling effect is different on them. Organics will wither instead. Organs will stop and limbs will become useless. Damage depends on amount of exposure, limited exposure will leave the tissue functional if scarred and crippled. Large exposure will leave tissue dead and after that it will crumble just like anything else.

  • Endless variety/numbers: There is no end in sight to the numbers of the monsters, society has basically just been holding them at bay at the frontier. They can come in many different shapes and sizes, appearing both like real animals and mythical ones.

  • No organs: Due to their nature as monsters of the void, they do not require anything to survive and therefore have no vital organs. Killing them is dependent on body damage. Cutting them to pieces and blowing them up are effective at killing them. Ranged weapons like guns generally don't do much more than slow them down. Although higher caliber and higher spread guns are effective. Large amounts of small wounds have also been shown to kill them, as if they were bleeding out, although no blood has ever been observed from them.




I need to work out what tactics would be best against these monsters. Both on an organizational army wide scale and independent unit scale ie what training and equipment soldiers receive and what tactics the army uses.



EDIT:



Alright I'll give you some extra details and clear some things up.




  • Withering organics is not an extra thing that happens before crumbling to ash. Its just the same thing happening in a different way.

  • touching a single part of an object will not cause the entire thing to crumble, only the touched area. Parts surrounding it will weaken however, monsters must use repeated attacks and touches to destroy things. This means the size of an object matters, a bullets small size will lead to near immediate disintegration upon entering their body but a thick stone wall could last a siege.

  • Their disintegration ability is in effect over the entire body, touch it anywhere and you will crumble. They are immune to the effect themselves so no friendly fire.

  • As creatures of the void, they are not all physically there. Hence no need for organs and such. While they can be touched and killed, they do not fit in with the rest of nature. Basically eldritch monsters. They also appear to fade away into smoke upon death, leaving no bodies.

  • The durability of the object matters when crumbling. e.g. stone lasts better than clay. Metal weapons can survive a few hits(as long as the structure is still sound)

  • There are monsters capable of things like flight, swimming and burrowing through the ground. Their numbers vary. Tunneling monsters are uncommon but do exist, they are more common in areas with plenty to dig through like mountains. It takes a while for them to dig but still faster than any natural earth creature thanks to their disintegration. Flyers are common but still less than standard land creatures. Anti-air weaponry would be effective against them.
    Oceans and water bodies in monster territory tend to be teeming with them. Sailing in monster waters is effectively suicide since they can destroy the bottoms of ships. The only sailing activity is limited to a safe and secure bay which is kept clean of monsters as well as freshwater bodies deep with safe territory.

  • The lower density of liquids and gases leads to the disintegration being less useful. e.g. water will become lifeless and less buoyant while air becomes stale and thin.

  • Monsters can tone down their ability at will, this is mostly done for convenience though, not much use walking when the ground is gone.

  • The origin of the monsters is not known by society. It appears they just spawn within their own territories. Created by their god.

  • Monster strategies usually come down to either strong individuals or pack hunting. Packs will usually stick to monsters of similar body types but this is not guaranteed. Monsters can form large hordes of many types which will attack in waves. Their God does little to lead them past simply creating them.

  • Monster intelligence can vary, most are nothing more than primal creatures whose only instinct is to seek out magic and destroy. But some can rise higher to use strategy in combat or lead their fellows like an alpha. Some can even organize entire armies of monsters but these are rare and their tactics pretty simplistic outside of maneuvering.

  • A single type of monster has developed which is capable of mimicking humans and infiltrating society to bring it down from inside. Their existence is not known to humans so tactics don't really exist against them.

  • Humanities goals has basically just been survive for their entire history since the monsters first started showing up. All they have been able to do. While they have fallen back and reclaimed bits of the frontier many times, they have never really been able to advance much further. Doing something about them will be the main plot of the story but for now I want to know the tactics already in use.

  • Very frontlines forts can expect 2-5 attacks a week, they use the time between attacks to recover, repair and restock.

  • It is not uncommon for groups of monsters to make their way past the frontline, this is why its so important for extra fortifications behind them. So recorded monster has made it all the way to the kingdom through the entire frontier's defenses.

  • Society is geared towards supporting the frontier, with resources and troops being sent out as often as possible. Internal politics can occasionally decrease this supply(because politicians can be bickering idiots at times)

  • Runes can not be freestanding or created instantly. They must be drawn onto something. So good for reinforcing a wall or adding an extra effect to it, not so much for force fields. Using them in the defense also has the extra drawback of attracting more monsters

  • The monsters cover basically the entire world except for this kingdom. Their more dense around the kingdom itself since the barrens don't really have any magic to attract them. It is theorized that there may be other kingdoms out there which have managed to survive like them. But between the monsters and distance contact it not viable until the threat is dealt with.

  • Terrain will regenerate over time when magic is available. Ley lines will naturally create features of the terrain and said features will simultaneously reinforce the ley lines. E.g. rivers and ridges are more likely to end up running over ley lines. This means that within the barrens the monsters have destroyed most features of the landscape to weaken and destroy the ley lines. As long as the kingdom still stands, the ley lines of the world will continue to regenerate using them as a source while simultaneously being targeted by monsters.

  • If the monsters were successful in destroying all magic including ley lines on the planet, it would die and come apart. If every planet in a system lost its lines then its star would be turned into a black hole(systems and planets function as gigantic runes keeping the universe together). The God of the void aims to do this everywhere

  • The monsters were not always attacking. There was a period where there was a good god protecting them until he got preoccupied with keeping the universe together elsewhere. They had gotten to about bronze age tech before the monsters started appearing. Much of the world was swiftly lost and they retreated into the fortified kingdom.




Phew, the number of things you only think to add after posting, well here we go. Time to say if anyone points out more details I could add....










share|improve this question















For my story the inhabitants of the setting find themselves fighting against endless hordes of monsters. But these monsters have several traits that will require unique strategies against them which I am trying to work out.



The setting
First of all some background on the world they're living in. Its set in a large kingdom about half the size of western Europe and is surrounded by barrens. These barrens are infested with the monsters and are constantly attacking. The area in between the kingdom and barrens is known as the frontier. A massive stretch of land filled with walls, fortresses and other defenses. If one of them were to fall it would be a simple matter to fall back. The tech level is undecided. I was thinking late second industrial revolution but medieval is also a possibility.



Magic
The setting does have a magic system but it is not too relevant to the tactics. It is of a runic nature and by drawing on ley lines anythign within the parameters of the rune can be done. It is not that suitable for use by units and is usually reserved for reinforcing defenses or enhancing long range weaponry like artillery.



The Monsters
The monsters are the creatures of a God of the void. They desire to wipe out everything and they are created by this god for doing so. They have multiple traits that make them difficult to stop.




  • Attraction to magic: The world is held together by magic so they have an ability to seek it out and destroy it. Magic can be found in all living things, running through the landscape as ley lines and in runic circles made by mages. This draws them endlessly into the kingdom which still has functional magic thanks to its defense from the monsters.

  • Anti-material: Anything the monsters touch crumbles into dust/ash. This severely limits the effectiveness of different weapons. Melee weapons will wear out quickly in combat and armour will crumble after a single hit. This obviously makes ranged weapons a smarter option. Defenses like walls can have some effectiveness at holding them back depending on thickness. They will need to be constantly patched and repaired however with constant attacks. Due to the magic running within living creatures their crumbling effect is different on them. Organics will wither instead. Organs will stop and limbs will become useless. Damage depends on amount of exposure, limited exposure will leave the tissue functional if scarred and crippled. Large exposure will leave tissue dead and after that it will crumble just like anything else.

  • Endless variety/numbers: There is no end in sight to the numbers of the monsters, society has basically just been holding them at bay at the frontier. They can come in many different shapes and sizes, appearing both like real animals and mythical ones.

  • No organs: Due to their nature as monsters of the void, they do not require anything to survive and therefore have no vital organs. Killing them is dependent on body damage. Cutting them to pieces and blowing them up are effective at killing them. Ranged weapons like guns generally don't do much more than slow them down. Although higher caliber and higher spread guns are effective. Large amounts of small wounds have also been shown to kill them, as if they were bleeding out, although no blood has ever been observed from them.




I need to work out what tactics would be best against these monsters. Both on an organizational army wide scale and independent unit scale ie what training and equipment soldiers receive and what tactics the army uses.



EDIT:



Alright I'll give you some extra details and clear some things up.




  • Withering organics is not an extra thing that happens before crumbling to ash. Its just the same thing happening in a different way.

  • touching a single part of an object will not cause the entire thing to crumble, only the touched area. Parts surrounding it will weaken however, monsters must use repeated attacks and touches to destroy things. This means the size of an object matters, a bullets small size will lead to near immediate disintegration upon entering their body but a thick stone wall could last a siege.

  • Their disintegration ability is in effect over the entire body, touch it anywhere and you will crumble. They are immune to the effect themselves so no friendly fire.

  • As creatures of the void, they are not all physically there. Hence no need for organs and such. While they can be touched and killed, they do not fit in with the rest of nature. Basically eldritch monsters. They also appear to fade away into smoke upon death, leaving no bodies.

  • The durability of the object matters when crumbling. e.g. stone lasts better than clay. Metal weapons can survive a few hits(as long as the structure is still sound)

  • There are monsters capable of things like flight, swimming and burrowing through the ground. Their numbers vary. Tunneling monsters are uncommon but do exist, they are more common in areas with plenty to dig through like mountains. It takes a while for them to dig but still faster than any natural earth creature thanks to their disintegration. Flyers are common but still less than standard land creatures. Anti-air weaponry would be effective against them.
    Oceans and water bodies in monster territory tend to be teeming with them. Sailing in monster waters is effectively suicide since they can destroy the bottoms of ships. The only sailing activity is limited to a safe and secure bay which is kept clean of monsters as well as freshwater bodies deep with safe territory.

  • The lower density of liquids and gases leads to the disintegration being less useful. e.g. water will become lifeless and less buoyant while air becomes stale and thin.

  • Monsters can tone down their ability at will, this is mostly done for convenience though, not much use walking when the ground is gone.

  • The origin of the monsters is not known by society. It appears they just spawn within their own territories. Created by their god.

  • Monster strategies usually come down to either strong individuals or pack hunting. Packs will usually stick to monsters of similar body types but this is not guaranteed. Monsters can form large hordes of many types which will attack in waves. Their God does little to lead them past simply creating them.

  • Monster intelligence can vary, most are nothing more than primal creatures whose only instinct is to seek out magic and destroy. But some can rise higher to use strategy in combat or lead their fellows like an alpha. Some can even organize entire armies of monsters but these are rare and their tactics pretty simplistic outside of maneuvering.

  • A single type of monster has developed which is capable of mimicking humans and infiltrating society to bring it down from inside. Their existence is not known to humans so tactics don't really exist against them.

  • Humanities goals has basically just been survive for their entire history since the monsters first started showing up. All they have been able to do. While they have fallen back and reclaimed bits of the frontier many times, they have never really been able to advance much further. Doing something about them will be the main plot of the story but for now I want to know the tactics already in use.

  • Very frontlines forts can expect 2-5 attacks a week, they use the time between attacks to recover, repair and restock.

  • It is not uncommon for groups of monsters to make their way past the frontline, this is why its so important for extra fortifications behind them. So recorded monster has made it all the way to the kingdom through the entire frontier's defenses.

  • Society is geared towards supporting the frontier, with resources and troops being sent out as often as possible. Internal politics can occasionally decrease this supply(because politicians can be bickering idiots at times)

  • Runes can not be freestanding or created instantly. They must be drawn onto something. So good for reinforcing a wall or adding an extra effect to it, not so much for force fields. Using them in the defense also has the extra drawback of attracting more monsters

  • The monsters cover basically the entire world except for this kingdom. Their more dense around the kingdom itself since the barrens don't really have any magic to attract them. It is theorized that there may be other kingdoms out there which have managed to survive like them. But between the monsters and distance contact it not viable until the threat is dealt with.

  • Terrain will regenerate over time when magic is available. Ley lines will naturally create features of the terrain and said features will simultaneously reinforce the ley lines. E.g. rivers and ridges are more likely to end up running over ley lines. This means that within the barrens the monsters have destroyed most features of the landscape to weaken and destroy the ley lines. As long as the kingdom still stands, the ley lines of the world will continue to regenerate using them as a source while simultaneously being targeted by monsters.

  • If the monsters were successful in destroying all magic including ley lines on the planet, it would die and come apart. If every planet in a system lost its lines then its star would be turned into a black hole(systems and planets function as gigantic runes keeping the universe together). The God of the void aims to do this everywhere

  • The monsters were not always attacking. There was a period where there was a good god protecting them until he got preoccupied with keeping the universe together elsewhere. They had gotten to about bronze age tech before the monsters started appearing. Much of the world was swiftly lost and they retreated into the fortified kingdom.




Phew, the number of things you only think to add after posting, well here we go. Time to say if anyone points out more details I could add....







creature-design military-defense strategy






share|improve this question















share|improve this question













share|improve this question




share|improve this question








edited 2 days ago

























asked 2 days ago









ArcWraith

505316




505316












  • Comments are not for extended discussion; this conversation has been moved to chat.
    – James
    yesterday


















  • Comments are not for extended discussion; this conversation has been moved to chat.
    – James
    yesterday
















Comments are not for extended discussion; this conversation has been moved to chat.
– James
yesterday




Comments are not for extended discussion; this conversation has been moved to chat.
– James
yesterday










14 Answers
14






active

oldest

votes

















up vote
65
down vote













Use their attraction towards magic to drown them into poop



It's too exhausting to renew all these expensive defense mechanisms. There is one resource the kingdom will never run out of: poop.



Let your drowning tank refills itself, as monsters are irresistibly lured toward the huge pile of magical artifacts you placed at the bottom of it.



It is actually a win-win situation: unpleasant odors are evacuated as poop is now constantly vanishing due to contact with the monsters.



Latter in your story, it will be discovered that the god sending the monsters is a benevolent one, who found this creative way to stimulate the development of sewage systems among humans.



drawning_in_poop



Possible downside The kingdom is being steadily depleted in organic material, especially nitrogen, so you will need a solution for farming.






share|improve this answer



















  • 4




    This answer deserves a bounty.
    – Renan
    2 days ago






  • 8




    "So you're telling me that you created endless hordes of monsters that can wipe out all life on the planet......to get rid of poop?"
    – ArcWraith
    2 days ago






  • 20




    "Am I not a brilliant God?!"
    – ArcWraith
    2 days ago






  • 3




    Guess this void this god is from is actually a void of the bowels
    – nullpointer
    2 days ago






  • 3




    "they do not require anything to survive" with that added to the OP, this will no longer stop them... however, it is still a viable way to slow them down while you pick them off. But being one of the archers posted to actually kill them seems like a crabby job =P
    – Tezra
    2 days ago


















up vote
9
down vote













Questions like this are hard because the answer is always the same: "Find what works well, and do it as much as possible." Sadly, this puts us in opposition with you. Our job is to break your system. Whatever rules you put in place, my job is to find ways to bend and break them.



Accordingly, the first thing I'd do is start using weapons that are still living. Why let my weapon crumble when its lifeforce can "wither" instead. All it needs to do is stay functional until the enemy is gone. Likewise, I'd probably build forts out of living trees to form a useful barrier.



Other than that, the answer would be the same as it is in every case. Find what is effective against the creatures, and do it repeatedly. You haven't stated in the question what works against them, so I can't say what that is. But your people will need to find it, and do it.



Also: take the fight to them. You can't just "survive" forever. If you never gain any ground, you'll only ever lose ground. My recommendation, not knowing anything useful about the creatures: raise violent pets. Start raising animals that might fight back against the creatures if unleashed. If you can train them to do something which hurts the creatures, train them accordingly. If you're up against an endless horde, and you're only surviving, you're losing.



Oh yeah, one more trick for anyone who has played Starcraft: Float to the island.






share|improve this answer





















  • I just played through part of that video quick to see what you were referring to. Moving the liftable buildings is a possible terran strategy, the other player was zerg, so I don't get why they didn't just ferry their army over in lords. By the time you're losing bad enough to liftoff, generally moving won't win unless opponent is slow and/or not very skilled since they already have you on the run. Still, thanks for the laugh. I recently started going back through the game again from beginning of original. Its story is better and so much more intricate than other sci-fi faves like Star Wars.
    – Aaron
    2 days ago










  • Organic weaponry is not viable. Withering is not an extra defense on top of the crumbling. Its the same thing happening in a different way. Wooden trees are less durable than stone walls so would be weaker(I'll edit that into my question) and the only places with the time to grow their defenses would be the very interior of the frontier. Plus attack animals would be such incredible animal cruelty. Their death rate would be colossal since a single bite would be enough to kill them(even if they survive they wouldn't be able to eat with the jaw damage)
    – ArcWraith
    2 days ago










  • @Aaron Yes, the game is balanced to make sure that Terran doesn't have an advantage against a skilled opponent on any reasonable map by floating their base away. Of course we know nothing about how skilled the opponents are in this world. Lifting off was a known effective tactic in the silver leagues where players didn't know how to combat it.
    – Cort Ammon
    2 days ago










  • @ArcWraith So what you're saying is the entire section on organics in your question isn't important, other than from the animal cruelty perspective? Their bodies fare no better than any other object?
    – Cort Ammon
    2 days ago






  • 3




    ... how well does the ground fare? Maybe I don't have to take these creatures out. Maybe they'll all just fall towards the center of the earth and solve my problem for me!
    – Cort Ammon
    2 days ago


















up vote
9
down vote













The monsters are anti-material, so weapons would be extremely ineffective as each one would be single use. There is no way any economy could create weapons at this rate.



Simply patching your walls could work for some time, depending on how fast your walls disintegrate. This however would result in an ever growing group of monsters tearing down your walls and will prove unsustainable in the long run.



The only way to limit your resource usage is to make sure you are not attacked by these monsters. Hiding obviously is not feasible since they track down your magic. So that only leaves one option: prevent them from reaching you! You can do this with a water cannon as used by the police force in riot control. You could even add in abrasive components as done in watter jet cutting to not only push the monsters away, but also actively hurt and possible deter them.



As @Mr.Mindor brings up: if the water is destroyed by the monsters it would turn into hydrogen and oxygen, which would react in an exothermic reaction. Likely the monsters would choose to turn their powers off while being bombarded as a way of protecting themselves from a huge fireball. This in turn would open up the possibility of shooting them with arrows, spears and the like as the previously mentioned problem with these is no longer in effect.



water canon from https://en.wikipedia.org/wiki/Water_cannon



You could build such a water canon by utilizing a dam that collects water. If you build a single structure like the Hoover Dam and build an efficient pipe network with limited losses you should be able to cover quite a large area depending on the density of the monsters.



The water would be coming out at just under 2.2 MPa (using the Bernoulli equation). Which is about the same as that of the riot police*.




p = rho * g * h




The most modern water cannons use 20 l/s*. The Colorado river supplies 500 m^3/s* so this should be enough to continously supply




500.000/20 = 25.000




cannons, which I recon should be enough to hold off at least 5.000 monsters at any given moment if you take into account a 60% loss (at the high end) and aim two canons at each monster.



*All numbers are from wikipedia






share|improve this answer










New contributor




Nathan is a new contributor to this site. Take care in asking for clarification, commenting, and answering.
Check out our Code of Conduct.


















  • While a viable and creative idea. I don't see how they would be able to supply enough water cannons and water to protect the entire frontier against hordes of monsters
    – ArcWraith
    2 days ago










  • @ArcWraith That's a fair point. I'll adjust my answer.
    – Nathan
    2 days ago










  • @ArcWraith I think it should be feasible, as long as you have a big enough dam and a big enough river.
    – Nathan
    2 days ago










  • I was thinking water also. Moats are a done thing.
    – Willk
    2 days ago










  • @ArcWraith Nathan makes a great point, no civilization will be able to survive if the resources it produces are constantly thrown away. If it takes dozens of bullets or arrows to destroy a single one of these creatures, how much effort did it take to mine the metal, how much time to grow the trees? Even with the magical regeneration you describe, it is doubtful they will be able to keep up. cont...
    – Mr.Mindor
    2 days ago


















up vote
7
down vote













Much as I admire the sheer elegance of the sewage solution, I'd think the answer is obvious. Classic RPG tactics would suggest that if physical attacks don't work, use energy attacks. If your world is at industrial levels, use fire and lightning.



Arrows and bags of manure, soaked in pitch and set alight before launch have been around since the Bronze Age. Even if the bags and contents disintegrate on impact, the fire won't and should severely burn large numbers of monsters if you use flaming bags.



Steampunk era gives you both static and current electricity. Connect your walls to giant van de Graaff generators, then sit back and watch the monsters get electrocuted as they touch the conductive plates. Set up large capacitor arrays and use them to power arrow fired tasers to incinerate large targets.



Since your monsters nullify, if not eat magic, you can't use mages, but mad science is always available to pick up the slack. If the above options work, upgrade to lasers, railguns and death rays.



EDIT: For better results, incorporate @Alexis' solution. Put cheap magical items in your flaming loads and hopefully, the monsters will actually lump together to grab it.






share|improve this answer



















  • 1




    Additionally, if you lob out two-part liquid explosives (i.e. when both parts mix, boom) and then put low-effort runes on the material separating the components... when the payload lands, if the separator breaks on impact it still explodes damaging any nearby monsters; if it doesn't, then monsters are drawn to the runes and when they disintegrate the separator, you get point-blank boom!
    – Doktor J
    yesterday


















up vote
4
down vote













Build walls and weapons out of materials that cannot wither or crumble to ash. A fortress in the frozen North could be built out of ice. Pikes could be gold coated. Burning oil doesn't wither nor does boiling tar.



In the end you need to destroy the portal letting them in. I'd build a fortress to hold them off and start a massive magical ritual for the purpose of attracting them as they seem to be creatures drive by instinct alone this way an attack force could attempt to close the portal from behind.






share|improve this answer




























    up vote
    4
    down vote













    Do different materials crumble into different types of dust/ash? If so, perhaps the defenders can find a material whose dust/ash has an angle of respose is so steep that the monsters can't climb it.



    Imagine trying to climb a mountain of powered sugar, you'd really just be wallowing in a pit of it before long, gradually eroding the mountain until the angle was such that you could actually approach.



    The defenders would need to maintain their mountain of powder, as the energy imparted by the monsters would distribute it. I imagine a bucket-brigade of peasants in snow-shows, raking the powder back into the correct slope so that it is ready to ensnare the next batch of monsters.



    Also, if the monsters are smart enough to build snow-shoes themselves, then this wont work.






    share|improve this answer




























      up vote
      4
      down vote













      The monsters are smoke. The god of the void is the fire.



      Your monsters are a symptom, like smoke is a symptom of fire. Smoke can definitely kill you and might be the worst thing about the fire, but it is difficult to fight smoke. You can avoid it or let it out or wear a mask but the smoke will keep coming, because the fire is still burning. To definitively fix a problem with smoke, you need to address the fire.



      And here the fire is the God of the void. Ideas with how to come to terms with a divine entity whose interests are contrary to yours.



      1: Come to terms. If the humans themselves facilitate the orderly destruction of magic items, that might be acceptable to the void god who would then be saved the trouble of making monsters. You would not want to destroy any faster than they can regenerate or this will not be sustainable/.



      2: Distract void god into doing something else elsewhere. That worked for the good god, and so there are presumably elsewheres accessable to gods. Point out to the void god that it will meet less resistance and be able to destroy magic faster in a place where humans are not opposing it.



      3: Recruit help. The existence of elsewheres and other gods suggest there might be other divine entities interested in preserving the status quo. One of these might be recruited to help humans oppose the void god as the good god used to - or maybe get that one back.



      4: Best defense is a good offense. There are gods and there are gods. Maybe you can dissuade the god of the void from his goal somehow. Remember he is a god; it will take more than one shot.






      share|improve this answer






























        up vote
        3
        down vote













        Deep trenches and moats would be useful for slowing down the monsters without presenting them with matter that would be vulnerable to their disintegration ability. It would also help group them so they would be more vulnerable to artillery and massed fire.



        Depending on the exact nature of your magic system - a magical barrier such as a runic force shield may be much more resistant to attack than any wall made of a mundane material. Reinforcing your structures with magic to resist disintegration may also work.



        As for offence - melee is obviously inadvisable, as is allowing them to get to close to your defences really. Massed artillery fire seems to be the way to hold them at bay, bolstered by magic as much as possible. Depending on your technology level, weapons such as white phosphorous will allow you to level the playing field.



        Flamethrowers could also allow small units to effectively combat the monsters, though fuel usage may be a problem.



        Rune circles could be used a magical "honeypots" to lure creatures into a certain area before shelling the ever-loving beejeezus out of it from a safe distance. If magic can affect the monstyers, laying down area affect spells to weaken them as they run toward you defences may also be useful






        share|improve this answer




























          up vote
          3
          down vote













          Play Tower Defense



          If you present them with a solid wall they will attack it and eventually break through, so instead you create a funnel and maze setup as a killing ground.



          The walls have openings, that they will naturally funnel through. Inside the walls are a maze or labyrinth that forces them to run around a very long route.



          While they do that towers and other placements along the walls fire arrows down into them until they are weakened and die. The towers are linked by mobile wooden bridges.



          If they do turn and try to attack the walls or towers then burning oil is dumped on them. If they begin to climb the wall or it becomes too weak the defenders retreat to the next one and pull up the bridges, leaving the creatures stranded on an island and forced to climb down into the maze again.






          share|improve this answer





















          • This doesn't address the fliers though, who would presumably attack the emplacements from above. Otherwise a good strategy!
            – Doktor J
            yesterday


















          up vote
          3
          down vote













          Material of choice: Clay



          With earth being plentiful, it should be easy to rapidly produce clay weapons and barriers, using runes to reinforce them as much as needed.



          Weapons of Choice: Heavy Bludgeoning/Crushing class weapons.



          The nature of the void creatures means that penetration (piercing/slashing) class weapons will have more surface area exposed to the monsters, and thus will wither significantly faster. Plus the creatures seem to be susceptible to "bleeding" effects. Crushing class weapons like the war-hammer are capable of causing massive internal bleeding without ever penetrating the target. This makes Bludgeoning weapons the most potent and durable weapons to use.



          For flying enemies, The requirements of flight already require a light (frail) body. Their void ability making the air thin will require an extra light frame to remain airborne. Any projectile should easily rip these enemies to shreds. So standard archers with longbows should do. (You may be able to modify them to fire clay bolts)



          For elite enemies, projectiles can be enhanced with Greek fire or gunpowder (or a runic emulation of the effect) for some extra kick.



          Armor of Choice: Leather



          This should be plentiful enough, and allow the soldiers to take a few hits, but the lightness/flexibility should help them to avoid getting hit in the first place. Note that this means soldiers will get well rounded physical training. They need endurance+agility to avoid getting hit, but they also need strength to maximize the damage of their heavy weapons.



          Attack Scheduling



          Either using caravans of magically reinforced elite troops, or routinely creating/destroying runic fortifications, this should allow the frontier forts to cycle which are the most "magically fortified", allowing the strongest forts to fight hardest while the neighbor forts use the relative downtime to repair/fortify their defenses; as well as cycle active duty troops so that those on the front lines are always at the top of their game.



          This bundling of enemies will also make siege class weapons like catapults more effective as their projectiles can hit more enemies per volley.



          Side note: The psychological health of our troops is important to remember. We will want to rotate troops in and out of the front regularly. And while resting, it's important that they have plenty of emotional support.



          Fort Style: Rocky terrain



          The endless nature of this enemy makes traditional munition based defense of traditional forts unsustainable. We need to get the enemy close enough for our troops to crush them, while creating a heavily obstacle ridden environment to choke/slow the enemy to deny them any swarm advantage. Catapults behind this fortification (or deep in the middle) can use their munition store to wipe out large groups or particularly strong looking enemies before they reach the main line. The nature of this defense also means it will remain largely intact if we are forced to retreat. This means that our forts will be up and running the moment we take it back from the enemy.






          share|improve this answer




























            up vote
            2
            down vote













            Fire.



            Nothing about these monsters seems to indicate that they can't burn.



            The disintegration effect is less useful against things of lower density, so, petrol seems like a solid go-to delivery mechanism, but really what you'd want to go for is the highest possible temperature, so any good raging inferno should do.



            You were less descriptive about the limitations of what runes can and can't do, but if they can produce fire or heat, it seems like making a reinforced runic pit of endless flame would be a pretty awesome defensive mechanism.






            share|improve this answer





















            • Oils (such as those rendered from animal fats or vegetables) have a double bonus -- not only are they of lower density, they're also organic in nature. They also burn quite well!
              – Doktor J
              yesterday


















            up vote
            1
            down vote













            Use their power against them



            Section removed as OP updated they are immune to their own anti-material. See edit history for how that could have been done otherwise.



            A more mundane approach



            Ranged weapons, whether by length or by projectile, are what started to win wars later on anyway, and many melee weapons are lethal from just 1 or 2 hits. So really, although these enemies sound awful when described, they really aren't individually a lot worse than fighting other humans after all.



            After re-reading the question again, I notice you suggest that arrows and guns just slow them down and it takes a lot of shots to kill them. In that case, arrows are less feasible in large scale attacks, but pole arms defending walls still work well. And when on the offensive, pole-arms behind trenches.



            Also start using modern poaching techniques. Leave lots of hidden traps for them to step on so they come at your slower and weakened.



            However, you did say there were huge numbers of them. That is probably the real problem here. But that is what castles are for. A castle allows its occupants to defend against a numerically vastly superior force, so again this just lends itself to what we see in our real history. However, if you rule that their touch can bring down walls too, then that might be problematic. However, some forts use earthwork fortifications, and disintegrating earth is not going to do much.



            Summary



            In a fantasy story, you can make their abilities sound awful, but in reality, against an army it isn't really all that bad and normal tactics which have been used in history can be used for the most part.



            Since that is not so exciting, as the author you should find the exciting and dramatic middle ground and just rule that it works out however you want. The people don't really need anything special to overcome these enemies, as I stated, but you can just make the enemies seem powerful and go whatever direction you feel like.






            share|improve this answer























            • Ok some specifics on how their anti-material ability works(I'll edit this into the question as well). They are immune to it so you can't use them against each other. Its a full body effect including internals not just hands. They cannot be starved(its states in the question they need nothing to survive). And while an amusing mental image, capturing them and putting them on poles is so not viable, since they will simply disintegrate the ropes. They also disappear on death.
              – ArcWraith
              2 days ago


















            up vote
            1
            down vote













            So the best way for you to defend against the monsters forever is to take advantage of your naturally generating terrain. This means that your weapons should reply on regenerated materials, rather then complex processed materials. (I am approaching this from the same way I played a infamous game called Dwarf Fortress)



            Defense 1 The Terrain



            The biggest advantage you have will be the terrain. A close source of Lava or Water would be invaluable as it would allow you to harness this magical liquid to fight the beasts. The easiest way is to simply build a huge dam that holds all this liquid and release it into the oncoming horde every time they attack. This lets you simply wipe some of them away and slow their progress as they now have to walk through muddy ground while fighting the current from the burst dam. After the breach, you refill the dam using the natural regeneration of the terrain or allow the water/lava to slowly pour in. Depending on the source, you could have multiple dams or only use a limited amount to help delay the beasts while your other measures kick in.



            You also want to limit where the monsters can approach from. Think of something like Helms deep from LOTR. Sure the monsters could approach from other directions, but they would never be able to do so in the numbers required to overtake your frontier forts (unless for some plot twist reason, you want to make it happen). Now the monsters might dig away at the sides and open up some passage ways, but when this happens you can send out some ranged forces to force them back and let it naturally regenerate over time.



            Defense 2 Stone Catapults and Ballista



            Terrain features regenerate overtime and there is always a abundance of stone to be found everywhere. Your siege weapons use this as a near infinite ammo. Not only will it regenerate over time providing you with a nearly unlimited source, its annoying for the monsters to remove. The battlie field will be littered with boulders and spears made out of stones which the monsters will have to push through. Combined with the a dam, you have a landslide that you can trigger once a battle. The monsters run in, you activate your dam. The boulders and spears from the previous battle are washed into the monsters and once the initial blast is over, you start firing, preparing the battlefield for the next battle.



            Defense 3 Rune enforce everything



            Runes stand out and attract more monsters. You want this, because you want to force your monsters to attack a fortress, not run around it. By providing a powerful enough incentive, you basically allowing the monsters to approach you the way you want them to. Reinforce all your defensive structures and watch the monsters pour in only to be washed out again. With the natural terrain supporting you, the monsters are going to find it extremely difficult to overcome you.






            share|improve this answer





















            • Rune enforcing everything is a good point to make. Focusing the monsters on the area most able to take them is definitely a good idea. As for the regenerating terrain, note the time it takes is highly dependent on scale. A simple rock on the side of the road could turn up in a month but a mountain would take centuries. Its also dependent on the position of the ley lines. An area without them would stay the same. The regeneration is also more dependant on natural processes. Erosion/chemical changes and the like would be influenced in the direction of ley with only minor matter creation
              – ArcWraith
              2 days ago












            • Sir, I've come to inform you that trebuchets are the superior seige engine.
              – tox123
              yesterday










            • @tox123 Even when trying to hit targets in the air?
              – Shadowzee
              yesterday










            • @Shadowzee with enough time, effort, dedication, and practice, yes.
              – tox123
              yesterday


















            up vote
            1
            down vote














            Cutting them to pieces and blowing them up are effective at killing
            them




            If available fragmentation grenades.



            If not whatever explosive you got combined with anything sharp. Nails, rocks with sharp edges, glass fragments.



            Very simple to build and cuts them up badly and inflicts a lot of damage which is why terrorist use them.



            You could add poison if it had any effect on them.






            share|improve this answer





















              Your Answer





              StackExchange.ifUsing("editor", function () {
              return StackExchange.using("mathjaxEditing", function () {
              StackExchange.MarkdownEditor.creationCallbacks.add(function (editor, postfix) {
              StackExchange.mathjaxEditing.prepareWmdForMathJax(editor, postfix, [["$", "$"], ["\\(","\\)"]]);
              });
              });
              }, "mathjax-editing");

              StackExchange.ready(function() {
              var channelOptions = {
              tags: "".split(" "),
              id: "579"
              };
              initTagRenderer("".split(" "), "".split(" "), channelOptions);

              StackExchange.using("externalEditor", function() {
              // Have to fire editor after snippets, if snippets enabled
              if (StackExchange.settings.snippets.snippetsEnabled) {
              StackExchange.using("snippets", function() {
              createEditor();
              });
              }
              else {
              createEditor();
              }
              });

              function createEditor() {
              StackExchange.prepareEditor({
              heartbeatType: 'answer',
              convertImagesToLinks: false,
              noModals: true,
              showLowRepImageUploadWarning: true,
              reputationToPostImages: null,
              bindNavPrevention: true,
              postfix: "",
              imageUploader: {
              brandingHtml: "Powered by u003ca class="icon-imgur-white" href="https://imgur.com/"u003eu003c/au003e",
              contentPolicyHtml: "User contributions licensed under u003ca href="https://creativecommons.org/licenses/by-sa/3.0/"u003ecc by-sa 3.0 with attribution requiredu003c/au003e u003ca href="https://stackoverflow.com/legal/content-policy"u003e(content policy)u003c/au003e",
              allowUrls: true
              },
              noCode: true, onDemand: true,
              discardSelector: ".discard-answer"
              ,immediatelyShowMarkdownHelp:true
              });


              }
              });














               

              draft saved


              draft discarded


















              StackExchange.ready(
              function () {
              StackExchange.openid.initPostLogin('.new-post-login', 'https%3a%2f%2fworldbuilding.stackexchange.com%2fquestions%2f129536%2fbest-strategy-against-anti-material-monsters%23new-answer', 'question_page');
              }
              );

              Post as a guest
































              14 Answers
              14






              active

              oldest

              votes








              14 Answers
              14






              active

              oldest

              votes









              active

              oldest

              votes






              active

              oldest

              votes








              up vote
              65
              down vote













              Use their attraction towards magic to drown them into poop



              It's too exhausting to renew all these expensive defense mechanisms. There is one resource the kingdom will never run out of: poop.



              Let your drowning tank refills itself, as monsters are irresistibly lured toward the huge pile of magical artifacts you placed at the bottom of it.



              It is actually a win-win situation: unpleasant odors are evacuated as poop is now constantly vanishing due to contact with the monsters.



              Latter in your story, it will be discovered that the god sending the monsters is a benevolent one, who found this creative way to stimulate the development of sewage systems among humans.



              drawning_in_poop



              Possible downside The kingdom is being steadily depleted in organic material, especially nitrogen, so you will need a solution for farming.






              share|improve this answer



















              • 4




                This answer deserves a bounty.
                – Renan
                2 days ago






              • 8




                "So you're telling me that you created endless hordes of monsters that can wipe out all life on the planet......to get rid of poop?"
                – ArcWraith
                2 days ago






              • 20




                "Am I not a brilliant God?!"
                – ArcWraith
                2 days ago






              • 3




                Guess this void this god is from is actually a void of the bowels
                – nullpointer
                2 days ago






              • 3




                "they do not require anything to survive" with that added to the OP, this will no longer stop them... however, it is still a viable way to slow them down while you pick them off. But being one of the archers posted to actually kill them seems like a crabby job =P
                – Tezra
                2 days ago















              up vote
              65
              down vote













              Use their attraction towards magic to drown them into poop



              It's too exhausting to renew all these expensive defense mechanisms. There is one resource the kingdom will never run out of: poop.



              Let your drowning tank refills itself, as monsters are irresistibly lured toward the huge pile of magical artifacts you placed at the bottom of it.



              It is actually a win-win situation: unpleasant odors are evacuated as poop is now constantly vanishing due to contact with the monsters.



              Latter in your story, it will be discovered that the god sending the monsters is a benevolent one, who found this creative way to stimulate the development of sewage systems among humans.



              drawning_in_poop



              Possible downside The kingdom is being steadily depleted in organic material, especially nitrogen, so you will need a solution for farming.






              share|improve this answer



















              • 4




                This answer deserves a bounty.
                – Renan
                2 days ago






              • 8




                "So you're telling me that you created endless hordes of monsters that can wipe out all life on the planet......to get rid of poop?"
                – ArcWraith
                2 days ago






              • 20




                "Am I not a brilliant God?!"
                – ArcWraith
                2 days ago






              • 3




                Guess this void this god is from is actually a void of the bowels
                – nullpointer
                2 days ago






              • 3




                "they do not require anything to survive" with that added to the OP, this will no longer stop them... however, it is still a viable way to slow them down while you pick them off. But being one of the archers posted to actually kill them seems like a crabby job =P
                – Tezra
                2 days ago













              up vote
              65
              down vote










              up vote
              65
              down vote









              Use their attraction towards magic to drown them into poop



              It's too exhausting to renew all these expensive defense mechanisms. There is one resource the kingdom will never run out of: poop.



              Let your drowning tank refills itself, as monsters are irresistibly lured toward the huge pile of magical artifacts you placed at the bottom of it.



              It is actually a win-win situation: unpleasant odors are evacuated as poop is now constantly vanishing due to contact with the monsters.



              Latter in your story, it will be discovered that the god sending the monsters is a benevolent one, who found this creative way to stimulate the development of sewage systems among humans.



              drawning_in_poop



              Possible downside The kingdom is being steadily depleted in organic material, especially nitrogen, so you will need a solution for farming.






              share|improve this answer














              Use their attraction towards magic to drown them into poop



              It's too exhausting to renew all these expensive defense mechanisms. There is one resource the kingdom will never run out of: poop.



              Let your drowning tank refills itself, as monsters are irresistibly lured toward the huge pile of magical artifacts you placed at the bottom of it.



              It is actually a win-win situation: unpleasant odors are evacuated as poop is now constantly vanishing due to contact with the monsters.



              Latter in your story, it will be discovered that the god sending the monsters is a benevolent one, who found this creative way to stimulate the development of sewage systems among humans.



              drawning_in_poop



              Possible downside The kingdom is being steadily depleted in organic material, especially nitrogen, so you will need a solution for farming.







              share|improve this answer














              share|improve this answer



              share|improve this answer








              edited 2 days ago

























              answered 2 days ago









              Alexis

              993318




              993318








              • 4




                This answer deserves a bounty.
                – Renan
                2 days ago






              • 8




                "So you're telling me that you created endless hordes of monsters that can wipe out all life on the planet......to get rid of poop?"
                – ArcWraith
                2 days ago






              • 20




                "Am I not a brilliant God?!"
                – ArcWraith
                2 days ago






              • 3




                Guess this void this god is from is actually a void of the bowels
                – nullpointer
                2 days ago






              • 3




                "they do not require anything to survive" with that added to the OP, this will no longer stop them... however, it is still a viable way to slow them down while you pick them off. But being one of the archers posted to actually kill them seems like a crabby job =P
                – Tezra
                2 days ago














              • 4




                This answer deserves a bounty.
                – Renan
                2 days ago






              • 8




                "So you're telling me that you created endless hordes of monsters that can wipe out all life on the planet......to get rid of poop?"
                – ArcWraith
                2 days ago






              • 20




                "Am I not a brilliant God?!"
                – ArcWraith
                2 days ago






              • 3




                Guess this void this god is from is actually a void of the bowels
                – nullpointer
                2 days ago






              • 3




                "they do not require anything to survive" with that added to the OP, this will no longer stop them... however, it is still a viable way to slow them down while you pick them off. But being one of the archers posted to actually kill them seems like a crabby job =P
                – Tezra
                2 days ago








              4




              4




              This answer deserves a bounty.
              – Renan
              2 days ago




              This answer deserves a bounty.
              – Renan
              2 days ago




              8




              8




              "So you're telling me that you created endless hordes of monsters that can wipe out all life on the planet......to get rid of poop?"
              – ArcWraith
              2 days ago




              "So you're telling me that you created endless hordes of monsters that can wipe out all life on the planet......to get rid of poop?"
              – ArcWraith
              2 days ago




              20




              20




              "Am I not a brilliant God?!"
              – ArcWraith
              2 days ago




              "Am I not a brilliant God?!"
              – ArcWraith
              2 days ago




              3




              3




              Guess this void this god is from is actually a void of the bowels
              – nullpointer
              2 days ago




              Guess this void this god is from is actually a void of the bowels
              – nullpointer
              2 days ago




              3




              3




              "they do not require anything to survive" with that added to the OP, this will no longer stop them... however, it is still a viable way to slow them down while you pick them off. But being one of the archers posted to actually kill them seems like a crabby job =P
              – Tezra
              2 days ago




              "they do not require anything to survive" with that added to the OP, this will no longer stop them... however, it is still a viable way to slow them down while you pick them off. But being one of the archers posted to actually kill them seems like a crabby job =P
              – Tezra
              2 days ago










              up vote
              9
              down vote













              Questions like this are hard because the answer is always the same: "Find what works well, and do it as much as possible." Sadly, this puts us in opposition with you. Our job is to break your system. Whatever rules you put in place, my job is to find ways to bend and break them.



              Accordingly, the first thing I'd do is start using weapons that are still living. Why let my weapon crumble when its lifeforce can "wither" instead. All it needs to do is stay functional until the enemy is gone. Likewise, I'd probably build forts out of living trees to form a useful barrier.



              Other than that, the answer would be the same as it is in every case. Find what is effective against the creatures, and do it repeatedly. You haven't stated in the question what works against them, so I can't say what that is. But your people will need to find it, and do it.



              Also: take the fight to them. You can't just "survive" forever. If you never gain any ground, you'll only ever lose ground. My recommendation, not knowing anything useful about the creatures: raise violent pets. Start raising animals that might fight back against the creatures if unleashed. If you can train them to do something which hurts the creatures, train them accordingly. If you're up against an endless horde, and you're only surviving, you're losing.



              Oh yeah, one more trick for anyone who has played Starcraft: Float to the island.






              share|improve this answer





















              • I just played through part of that video quick to see what you were referring to. Moving the liftable buildings is a possible terran strategy, the other player was zerg, so I don't get why they didn't just ferry their army over in lords. By the time you're losing bad enough to liftoff, generally moving won't win unless opponent is slow and/or not very skilled since they already have you on the run. Still, thanks for the laugh. I recently started going back through the game again from beginning of original. Its story is better and so much more intricate than other sci-fi faves like Star Wars.
                – Aaron
                2 days ago










              • Organic weaponry is not viable. Withering is not an extra defense on top of the crumbling. Its the same thing happening in a different way. Wooden trees are less durable than stone walls so would be weaker(I'll edit that into my question) and the only places with the time to grow their defenses would be the very interior of the frontier. Plus attack animals would be such incredible animal cruelty. Their death rate would be colossal since a single bite would be enough to kill them(even if they survive they wouldn't be able to eat with the jaw damage)
                – ArcWraith
                2 days ago










              • @Aaron Yes, the game is balanced to make sure that Terran doesn't have an advantage against a skilled opponent on any reasonable map by floating their base away. Of course we know nothing about how skilled the opponents are in this world. Lifting off was a known effective tactic in the silver leagues where players didn't know how to combat it.
                – Cort Ammon
                2 days ago










              • @ArcWraith So what you're saying is the entire section on organics in your question isn't important, other than from the animal cruelty perspective? Their bodies fare no better than any other object?
                – Cort Ammon
                2 days ago






              • 3




                ... how well does the ground fare? Maybe I don't have to take these creatures out. Maybe they'll all just fall towards the center of the earth and solve my problem for me!
                – Cort Ammon
                2 days ago















              up vote
              9
              down vote













              Questions like this are hard because the answer is always the same: "Find what works well, and do it as much as possible." Sadly, this puts us in opposition with you. Our job is to break your system. Whatever rules you put in place, my job is to find ways to bend and break them.



              Accordingly, the first thing I'd do is start using weapons that are still living. Why let my weapon crumble when its lifeforce can "wither" instead. All it needs to do is stay functional until the enemy is gone. Likewise, I'd probably build forts out of living trees to form a useful barrier.



              Other than that, the answer would be the same as it is in every case. Find what is effective against the creatures, and do it repeatedly. You haven't stated in the question what works against them, so I can't say what that is. But your people will need to find it, and do it.



              Also: take the fight to them. You can't just "survive" forever. If you never gain any ground, you'll only ever lose ground. My recommendation, not knowing anything useful about the creatures: raise violent pets. Start raising animals that might fight back against the creatures if unleashed. If you can train them to do something which hurts the creatures, train them accordingly. If you're up against an endless horde, and you're only surviving, you're losing.



              Oh yeah, one more trick for anyone who has played Starcraft: Float to the island.






              share|improve this answer





















              • I just played through part of that video quick to see what you were referring to. Moving the liftable buildings is a possible terran strategy, the other player was zerg, so I don't get why they didn't just ferry their army over in lords. By the time you're losing bad enough to liftoff, generally moving won't win unless opponent is slow and/or not very skilled since they already have you on the run. Still, thanks for the laugh. I recently started going back through the game again from beginning of original. Its story is better and so much more intricate than other sci-fi faves like Star Wars.
                – Aaron
                2 days ago










              • Organic weaponry is not viable. Withering is not an extra defense on top of the crumbling. Its the same thing happening in a different way. Wooden trees are less durable than stone walls so would be weaker(I'll edit that into my question) and the only places with the time to grow their defenses would be the very interior of the frontier. Plus attack animals would be such incredible animal cruelty. Their death rate would be colossal since a single bite would be enough to kill them(even if they survive they wouldn't be able to eat with the jaw damage)
                – ArcWraith
                2 days ago










              • @Aaron Yes, the game is balanced to make sure that Terran doesn't have an advantage against a skilled opponent on any reasonable map by floating their base away. Of course we know nothing about how skilled the opponents are in this world. Lifting off was a known effective tactic in the silver leagues where players didn't know how to combat it.
                – Cort Ammon
                2 days ago










              • @ArcWraith So what you're saying is the entire section on organics in your question isn't important, other than from the animal cruelty perspective? Their bodies fare no better than any other object?
                – Cort Ammon
                2 days ago






              • 3




                ... how well does the ground fare? Maybe I don't have to take these creatures out. Maybe they'll all just fall towards the center of the earth and solve my problem for me!
                – Cort Ammon
                2 days ago













              up vote
              9
              down vote










              up vote
              9
              down vote









              Questions like this are hard because the answer is always the same: "Find what works well, and do it as much as possible." Sadly, this puts us in opposition with you. Our job is to break your system. Whatever rules you put in place, my job is to find ways to bend and break them.



              Accordingly, the first thing I'd do is start using weapons that are still living. Why let my weapon crumble when its lifeforce can "wither" instead. All it needs to do is stay functional until the enemy is gone. Likewise, I'd probably build forts out of living trees to form a useful barrier.



              Other than that, the answer would be the same as it is in every case. Find what is effective against the creatures, and do it repeatedly. You haven't stated in the question what works against them, so I can't say what that is. But your people will need to find it, and do it.



              Also: take the fight to them. You can't just "survive" forever. If you never gain any ground, you'll only ever lose ground. My recommendation, not knowing anything useful about the creatures: raise violent pets. Start raising animals that might fight back against the creatures if unleashed. If you can train them to do something which hurts the creatures, train them accordingly. If you're up against an endless horde, and you're only surviving, you're losing.



              Oh yeah, one more trick for anyone who has played Starcraft: Float to the island.






              share|improve this answer












              Questions like this are hard because the answer is always the same: "Find what works well, and do it as much as possible." Sadly, this puts us in opposition with you. Our job is to break your system. Whatever rules you put in place, my job is to find ways to bend and break them.



              Accordingly, the first thing I'd do is start using weapons that are still living. Why let my weapon crumble when its lifeforce can "wither" instead. All it needs to do is stay functional until the enemy is gone. Likewise, I'd probably build forts out of living trees to form a useful barrier.



              Other than that, the answer would be the same as it is in every case. Find what is effective against the creatures, and do it repeatedly. You haven't stated in the question what works against them, so I can't say what that is. But your people will need to find it, and do it.



              Also: take the fight to them. You can't just "survive" forever. If you never gain any ground, you'll only ever lose ground. My recommendation, not knowing anything useful about the creatures: raise violent pets. Start raising animals that might fight back against the creatures if unleashed. If you can train them to do something which hurts the creatures, train them accordingly. If you're up against an endless horde, and you're only surviving, you're losing.



              Oh yeah, one more trick for anyone who has played Starcraft: Float to the island.







              share|improve this answer












              share|improve this answer



              share|improve this answer










              answered 2 days ago









              Cort Ammon

              104k15180369




              104k15180369












              • I just played through part of that video quick to see what you were referring to. Moving the liftable buildings is a possible terran strategy, the other player was zerg, so I don't get why they didn't just ferry their army over in lords. By the time you're losing bad enough to liftoff, generally moving won't win unless opponent is slow and/or not very skilled since they already have you on the run. Still, thanks for the laugh. I recently started going back through the game again from beginning of original. Its story is better and so much more intricate than other sci-fi faves like Star Wars.
                – Aaron
                2 days ago










              • Organic weaponry is not viable. Withering is not an extra defense on top of the crumbling. Its the same thing happening in a different way. Wooden trees are less durable than stone walls so would be weaker(I'll edit that into my question) and the only places with the time to grow their defenses would be the very interior of the frontier. Plus attack animals would be such incredible animal cruelty. Their death rate would be colossal since a single bite would be enough to kill them(even if they survive they wouldn't be able to eat with the jaw damage)
                – ArcWraith
                2 days ago










              • @Aaron Yes, the game is balanced to make sure that Terran doesn't have an advantage against a skilled opponent on any reasonable map by floating their base away. Of course we know nothing about how skilled the opponents are in this world. Lifting off was a known effective tactic in the silver leagues where players didn't know how to combat it.
                – Cort Ammon
                2 days ago










              • @ArcWraith So what you're saying is the entire section on organics in your question isn't important, other than from the animal cruelty perspective? Their bodies fare no better than any other object?
                – Cort Ammon
                2 days ago






              • 3




                ... how well does the ground fare? Maybe I don't have to take these creatures out. Maybe they'll all just fall towards the center of the earth and solve my problem for me!
                – Cort Ammon
                2 days ago


















              • I just played through part of that video quick to see what you were referring to. Moving the liftable buildings is a possible terran strategy, the other player was zerg, so I don't get why they didn't just ferry their army over in lords. By the time you're losing bad enough to liftoff, generally moving won't win unless opponent is slow and/or not very skilled since they already have you on the run. Still, thanks for the laugh. I recently started going back through the game again from beginning of original. Its story is better and so much more intricate than other sci-fi faves like Star Wars.
                – Aaron
                2 days ago










              • Organic weaponry is not viable. Withering is not an extra defense on top of the crumbling. Its the same thing happening in a different way. Wooden trees are less durable than stone walls so would be weaker(I'll edit that into my question) and the only places with the time to grow their defenses would be the very interior of the frontier. Plus attack animals would be such incredible animal cruelty. Their death rate would be colossal since a single bite would be enough to kill them(even if they survive they wouldn't be able to eat with the jaw damage)
                – ArcWraith
                2 days ago










              • @Aaron Yes, the game is balanced to make sure that Terran doesn't have an advantage against a skilled opponent on any reasonable map by floating their base away. Of course we know nothing about how skilled the opponents are in this world. Lifting off was a known effective tactic in the silver leagues where players didn't know how to combat it.
                – Cort Ammon
                2 days ago










              • @ArcWraith So what you're saying is the entire section on organics in your question isn't important, other than from the animal cruelty perspective? Their bodies fare no better than any other object?
                – Cort Ammon
                2 days ago






              • 3




                ... how well does the ground fare? Maybe I don't have to take these creatures out. Maybe they'll all just fall towards the center of the earth and solve my problem for me!
                – Cort Ammon
                2 days ago
















              I just played through part of that video quick to see what you were referring to. Moving the liftable buildings is a possible terran strategy, the other player was zerg, so I don't get why they didn't just ferry their army over in lords. By the time you're losing bad enough to liftoff, generally moving won't win unless opponent is slow and/or not very skilled since they already have you on the run. Still, thanks for the laugh. I recently started going back through the game again from beginning of original. Its story is better and so much more intricate than other sci-fi faves like Star Wars.
              – Aaron
              2 days ago




              I just played through part of that video quick to see what you were referring to. Moving the liftable buildings is a possible terran strategy, the other player was zerg, so I don't get why they didn't just ferry their army over in lords. By the time you're losing bad enough to liftoff, generally moving won't win unless opponent is slow and/or not very skilled since they already have you on the run. Still, thanks for the laugh. I recently started going back through the game again from beginning of original. Its story is better and so much more intricate than other sci-fi faves like Star Wars.
              – Aaron
              2 days ago












              Organic weaponry is not viable. Withering is not an extra defense on top of the crumbling. Its the same thing happening in a different way. Wooden trees are less durable than stone walls so would be weaker(I'll edit that into my question) and the only places with the time to grow their defenses would be the very interior of the frontier. Plus attack animals would be such incredible animal cruelty. Their death rate would be colossal since a single bite would be enough to kill them(even if they survive they wouldn't be able to eat with the jaw damage)
              – ArcWraith
              2 days ago




              Organic weaponry is not viable. Withering is not an extra defense on top of the crumbling. Its the same thing happening in a different way. Wooden trees are less durable than stone walls so would be weaker(I'll edit that into my question) and the only places with the time to grow their defenses would be the very interior of the frontier. Plus attack animals would be such incredible animal cruelty. Their death rate would be colossal since a single bite would be enough to kill them(even if they survive they wouldn't be able to eat with the jaw damage)
              – ArcWraith
              2 days ago












              @Aaron Yes, the game is balanced to make sure that Terran doesn't have an advantage against a skilled opponent on any reasonable map by floating their base away. Of course we know nothing about how skilled the opponents are in this world. Lifting off was a known effective tactic in the silver leagues where players didn't know how to combat it.
              – Cort Ammon
              2 days ago




              @Aaron Yes, the game is balanced to make sure that Terran doesn't have an advantage against a skilled opponent on any reasonable map by floating their base away. Of course we know nothing about how skilled the opponents are in this world. Lifting off was a known effective tactic in the silver leagues where players didn't know how to combat it.
              – Cort Ammon
              2 days ago












              @ArcWraith So what you're saying is the entire section on organics in your question isn't important, other than from the animal cruelty perspective? Their bodies fare no better than any other object?
              – Cort Ammon
              2 days ago




              @ArcWraith So what you're saying is the entire section on organics in your question isn't important, other than from the animal cruelty perspective? Their bodies fare no better than any other object?
              – Cort Ammon
              2 days ago




              3




              3




              ... how well does the ground fare? Maybe I don't have to take these creatures out. Maybe they'll all just fall towards the center of the earth and solve my problem for me!
              – Cort Ammon
              2 days ago




              ... how well does the ground fare? Maybe I don't have to take these creatures out. Maybe they'll all just fall towards the center of the earth and solve my problem for me!
              – Cort Ammon
              2 days ago










              up vote
              9
              down vote













              The monsters are anti-material, so weapons would be extremely ineffective as each one would be single use. There is no way any economy could create weapons at this rate.



              Simply patching your walls could work for some time, depending on how fast your walls disintegrate. This however would result in an ever growing group of monsters tearing down your walls and will prove unsustainable in the long run.



              The only way to limit your resource usage is to make sure you are not attacked by these monsters. Hiding obviously is not feasible since they track down your magic. So that only leaves one option: prevent them from reaching you! You can do this with a water cannon as used by the police force in riot control. You could even add in abrasive components as done in watter jet cutting to not only push the monsters away, but also actively hurt and possible deter them.



              As @Mr.Mindor brings up: if the water is destroyed by the monsters it would turn into hydrogen and oxygen, which would react in an exothermic reaction. Likely the monsters would choose to turn their powers off while being bombarded as a way of protecting themselves from a huge fireball. This in turn would open up the possibility of shooting them with arrows, spears and the like as the previously mentioned problem with these is no longer in effect.



              water canon from https://en.wikipedia.org/wiki/Water_cannon



              You could build such a water canon by utilizing a dam that collects water. If you build a single structure like the Hoover Dam and build an efficient pipe network with limited losses you should be able to cover quite a large area depending on the density of the monsters.



              The water would be coming out at just under 2.2 MPa (using the Bernoulli equation). Which is about the same as that of the riot police*.




              p = rho * g * h




              The most modern water cannons use 20 l/s*. The Colorado river supplies 500 m^3/s* so this should be enough to continously supply




              500.000/20 = 25.000




              cannons, which I recon should be enough to hold off at least 5.000 monsters at any given moment if you take into account a 60% loss (at the high end) and aim two canons at each monster.



              *All numbers are from wikipedia






              share|improve this answer










              New contributor




              Nathan is a new contributor to this site. Take care in asking for clarification, commenting, and answering.
              Check out our Code of Conduct.


















              • While a viable and creative idea. I don't see how they would be able to supply enough water cannons and water to protect the entire frontier against hordes of monsters
                – ArcWraith
                2 days ago










              • @ArcWraith That's a fair point. I'll adjust my answer.
                – Nathan
                2 days ago










              • @ArcWraith I think it should be feasible, as long as you have a big enough dam and a big enough river.
                – Nathan
                2 days ago










              • I was thinking water also. Moats are a done thing.
                – Willk
                2 days ago










              • @ArcWraith Nathan makes a great point, no civilization will be able to survive if the resources it produces are constantly thrown away. If it takes dozens of bullets or arrows to destroy a single one of these creatures, how much effort did it take to mine the metal, how much time to grow the trees? Even with the magical regeneration you describe, it is doubtful they will be able to keep up. cont...
                – Mr.Mindor
                2 days ago















              up vote
              9
              down vote













              The monsters are anti-material, so weapons would be extremely ineffective as each one would be single use. There is no way any economy could create weapons at this rate.



              Simply patching your walls could work for some time, depending on how fast your walls disintegrate. This however would result in an ever growing group of monsters tearing down your walls and will prove unsustainable in the long run.



              The only way to limit your resource usage is to make sure you are not attacked by these monsters. Hiding obviously is not feasible since they track down your magic. So that only leaves one option: prevent them from reaching you! You can do this with a water cannon as used by the police force in riot control. You could even add in abrasive components as done in watter jet cutting to not only push the monsters away, but also actively hurt and possible deter them.



              As @Mr.Mindor brings up: if the water is destroyed by the monsters it would turn into hydrogen and oxygen, which would react in an exothermic reaction. Likely the monsters would choose to turn their powers off while being bombarded as a way of protecting themselves from a huge fireball. This in turn would open up the possibility of shooting them with arrows, spears and the like as the previously mentioned problem with these is no longer in effect.



              water canon from https://en.wikipedia.org/wiki/Water_cannon



              You could build such a water canon by utilizing a dam that collects water. If you build a single structure like the Hoover Dam and build an efficient pipe network with limited losses you should be able to cover quite a large area depending on the density of the monsters.



              The water would be coming out at just under 2.2 MPa (using the Bernoulli equation). Which is about the same as that of the riot police*.




              p = rho * g * h




              The most modern water cannons use 20 l/s*. The Colorado river supplies 500 m^3/s* so this should be enough to continously supply




              500.000/20 = 25.000




              cannons, which I recon should be enough to hold off at least 5.000 monsters at any given moment if you take into account a 60% loss (at the high end) and aim two canons at each monster.



              *All numbers are from wikipedia






              share|improve this answer










              New contributor




              Nathan is a new contributor to this site. Take care in asking for clarification, commenting, and answering.
              Check out our Code of Conduct.


















              • While a viable and creative idea. I don't see how they would be able to supply enough water cannons and water to protect the entire frontier against hordes of monsters
                – ArcWraith
                2 days ago










              • @ArcWraith That's a fair point. I'll adjust my answer.
                – Nathan
                2 days ago










              • @ArcWraith I think it should be feasible, as long as you have a big enough dam and a big enough river.
                – Nathan
                2 days ago










              • I was thinking water also. Moats are a done thing.
                – Willk
                2 days ago










              • @ArcWraith Nathan makes a great point, no civilization will be able to survive if the resources it produces are constantly thrown away. If it takes dozens of bullets or arrows to destroy a single one of these creatures, how much effort did it take to mine the metal, how much time to grow the trees? Even with the magical regeneration you describe, it is doubtful they will be able to keep up. cont...
                – Mr.Mindor
                2 days ago













              up vote
              9
              down vote










              up vote
              9
              down vote









              The monsters are anti-material, so weapons would be extremely ineffective as each one would be single use. There is no way any economy could create weapons at this rate.



              Simply patching your walls could work for some time, depending on how fast your walls disintegrate. This however would result in an ever growing group of monsters tearing down your walls and will prove unsustainable in the long run.



              The only way to limit your resource usage is to make sure you are not attacked by these monsters. Hiding obviously is not feasible since they track down your magic. So that only leaves one option: prevent them from reaching you! You can do this with a water cannon as used by the police force in riot control. You could even add in abrasive components as done in watter jet cutting to not only push the monsters away, but also actively hurt and possible deter them.



              As @Mr.Mindor brings up: if the water is destroyed by the monsters it would turn into hydrogen and oxygen, which would react in an exothermic reaction. Likely the monsters would choose to turn their powers off while being bombarded as a way of protecting themselves from a huge fireball. This in turn would open up the possibility of shooting them with arrows, spears and the like as the previously mentioned problem with these is no longer in effect.



              water canon from https://en.wikipedia.org/wiki/Water_cannon



              You could build such a water canon by utilizing a dam that collects water. If you build a single structure like the Hoover Dam and build an efficient pipe network with limited losses you should be able to cover quite a large area depending on the density of the monsters.



              The water would be coming out at just under 2.2 MPa (using the Bernoulli equation). Which is about the same as that of the riot police*.




              p = rho * g * h




              The most modern water cannons use 20 l/s*. The Colorado river supplies 500 m^3/s* so this should be enough to continously supply




              500.000/20 = 25.000




              cannons, which I recon should be enough to hold off at least 5.000 monsters at any given moment if you take into account a 60% loss (at the high end) and aim two canons at each monster.



              *All numbers are from wikipedia






              share|improve this answer










              New contributor




              Nathan is a new contributor to this site. Take care in asking for clarification, commenting, and answering.
              Check out our Code of Conduct.









              The monsters are anti-material, so weapons would be extremely ineffective as each one would be single use. There is no way any economy could create weapons at this rate.



              Simply patching your walls could work for some time, depending on how fast your walls disintegrate. This however would result in an ever growing group of monsters tearing down your walls and will prove unsustainable in the long run.



              The only way to limit your resource usage is to make sure you are not attacked by these monsters. Hiding obviously is not feasible since they track down your magic. So that only leaves one option: prevent them from reaching you! You can do this with a water cannon as used by the police force in riot control. You could even add in abrasive components as done in watter jet cutting to not only push the monsters away, but also actively hurt and possible deter them.



              As @Mr.Mindor brings up: if the water is destroyed by the monsters it would turn into hydrogen and oxygen, which would react in an exothermic reaction. Likely the monsters would choose to turn their powers off while being bombarded as a way of protecting themselves from a huge fireball. This in turn would open up the possibility of shooting them with arrows, spears and the like as the previously mentioned problem with these is no longer in effect.



              water canon from https://en.wikipedia.org/wiki/Water_cannon



              You could build such a water canon by utilizing a dam that collects water. If you build a single structure like the Hoover Dam and build an efficient pipe network with limited losses you should be able to cover quite a large area depending on the density of the monsters.



              The water would be coming out at just under 2.2 MPa (using the Bernoulli equation). Which is about the same as that of the riot police*.




              p = rho * g * h




              The most modern water cannons use 20 l/s*. The Colorado river supplies 500 m^3/s* so this should be enough to continously supply




              500.000/20 = 25.000




              cannons, which I recon should be enough to hold off at least 5.000 monsters at any given moment if you take into account a 60% loss (at the high end) and aim two canons at each monster.



              *All numbers are from wikipedia







              share|improve this answer










              New contributor




              Nathan is a new contributor to this site. Take care in asking for clarification, commenting, and answering.
              Check out our Code of Conduct.









              share|improve this answer



              share|improve this answer








              edited 2 days ago





















              New contributor




              Nathan is a new contributor to this site. Take care in asking for clarification, commenting, and answering.
              Check out our Code of Conduct.









              answered 2 days ago









              Nathan

              1914




              1914




              New contributor




              Nathan is a new contributor to this site. Take care in asking for clarification, commenting, and answering.
              Check out our Code of Conduct.





              New contributor





              Nathan is a new contributor to this site. Take care in asking for clarification, commenting, and answering.
              Check out our Code of Conduct.






              Nathan is a new contributor to this site. Take care in asking for clarification, commenting, and answering.
              Check out our Code of Conduct.












              • While a viable and creative idea. I don't see how they would be able to supply enough water cannons and water to protect the entire frontier against hordes of monsters
                – ArcWraith
                2 days ago










              • @ArcWraith That's a fair point. I'll adjust my answer.
                – Nathan
                2 days ago










              • @ArcWraith I think it should be feasible, as long as you have a big enough dam and a big enough river.
                – Nathan
                2 days ago










              • I was thinking water also. Moats are a done thing.
                – Willk
                2 days ago










              • @ArcWraith Nathan makes a great point, no civilization will be able to survive if the resources it produces are constantly thrown away. If it takes dozens of bullets or arrows to destroy a single one of these creatures, how much effort did it take to mine the metal, how much time to grow the trees? Even with the magical regeneration you describe, it is doubtful they will be able to keep up. cont...
                – Mr.Mindor
                2 days ago


















              • While a viable and creative idea. I don't see how they would be able to supply enough water cannons and water to protect the entire frontier against hordes of monsters
                – ArcWraith
                2 days ago










              • @ArcWraith That's a fair point. I'll adjust my answer.
                – Nathan
                2 days ago










              • @ArcWraith I think it should be feasible, as long as you have a big enough dam and a big enough river.
                – Nathan
                2 days ago










              • I was thinking water also. Moats are a done thing.
                – Willk
                2 days ago










              • @ArcWraith Nathan makes a great point, no civilization will be able to survive if the resources it produces are constantly thrown away. If it takes dozens of bullets or arrows to destroy a single one of these creatures, how much effort did it take to mine the metal, how much time to grow the trees? Even with the magical regeneration you describe, it is doubtful they will be able to keep up. cont...
                – Mr.Mindor
                2 days ago
















              While a viable and creative idea. I don't see how they would be able to supply enough water cannons and water to protect the entire frontier against hordes of monsters
              – ArcWraith
              2 days ago




              While a viable and creative idea. I don't see how they would be able to supply enough water cannons and water to protect the entire frontier against hordes of monsters
              – ArcWraith
              2 days ago












              @ArcWraith That's a fair point. I'll adjust my answer.
              – Nathan
              2 days ago




              @ArcWraith That's a fair point. I'll adjust my answer.
              – Nathan
              2 days ago












              @ArcWraith I think it should be feasible, as long as you have a big enough dam and a big enough river.
              – Nathan
              2 days ago




              @ArcWraith I think it should be feasible, as long as you have a big enough dam and a big enough river.
              – Nathan
              2 days ago












              I was thinking water also. Moats are a done thing.
              – Willk
              2 days ago




              I was thinking water also. Moats are a done thing.
              – Willk
              2 days ago












              @ArcWraith Nathan makes a great point, no civilization will be able to survive if the resources it produces are constantly thrown away. If it takes dozens of bullets or arrows to destroy a single one of these creatures, how much effort did it take to mine the metal, how much time to grow the trees? Even with the magical regeneration you describe, it is doubtful they will be able to keep up. cont...
              – Mr.Mindor
              2 days ago




              @ArcWraith Nathan makes a great point, no civilization will be able to survive if the resources it produces are constantly thrown away. If it takes dozens of bullets or arrows to destroy a single one of these creatures, how much effort did it take to mine the metal, how much time to grow the trees? Even with the magical regeneration you describe, it is doubtful they will be able to keep up. cont...
              – Mr.Mindor
              2 days ago










              up vote
              7
              down vote













              Much as I admire the sheer elegance of the sewage solution, I'd think the answer is obvious. Classic RPG tactics would suggest that if physical attacks don't work, use energy attacks. If your world is at industrial levels, use fire and lightning.



              Arrows and bags of manure, soaked in pitch and set alight before launch have been around since the Bronze Age. Even if the bags and contents disintegrate on impact, the fire won't and should severely burn large numbers of monsters if you use flaming bags.



              Steampunk era gives you both static and current electricity. Connect your walls to giant van de Graaff generators, then sit back and watch the monsters get electrocuted as they touch the conductive plates. Set up large capacitor arrays and use them to power arrow fired tasers to incinerate large targets.



              Since your monsters nullify, if not eat magic, you can't use mages, but mad science is always available to pick up the slack. If the above options work, upgrade to lasers, railguns and death rays.



              EDIT: For better results, incorporate @Alexis' solution. Put cheap magical items in your flaming loads and hopefully, the monsters will actually lump together to grab it.






              share|improve this answer



















              • 1




                Additionally, if you lob out two-part liquid explosives (i.e. when both parts mix, boom) and then put low-effort runes on the material separating the components... when the payload lands, if the separator breaks on impact it still explodes damaging any nearby monsters; if it doesn't, then monsters are drawn to the runes and when they disintegrate the separator, you get point-blank boom!
                – Doktor J
                yesterday















              up vote
              7
              down vote













              Much as I admire the sheer elegance of the sewage solution, I'd think the answer is obvious. Classic RPG tactics would suggest that if physical attacks don't work, use energy attacks. If your world is at industrial levels, use fire and lightning.



              Arrows and bags of manure, soaked in pitch and set alight before launch have been around since the Bronze Age. Even if the bags and contents disintegrate on impact, the fire won't and should severely burn large numbers of monsters if you use flaming bags.



              Steampunk era gives you both static and current electricity. Connect your walls to giant van de Graaff generators, then sit back and watch the monsters get electrocuted as they touch the conductive plates. Set up large capacitor arrays and use them to power arrow fired tasers to incinerate large targets.



              Since your monsters nullify, if not eat magic, you can't use mages, but mad science is always available to pick up the slack. If the above options work, upgrade to lasers, railguns and death rays.



              EDIT: For better results, incorporate @Alexis' solution. Put cheap magical items in your flaming loads and hopefully, the monsters will actually lump together to grab it.






              share|improve this answer



















              • 1




                Additionally, if you lob out two-part liquid explosives (i.e. when both parts mix, boom) and then put low-effort runes on the material separating the components... when the payload lands, if the separator breaks on impact it still explodes damaging any nearby monsters; if it doesn't, then monsters are drawn to the runes and when they disintegrate the separator, you get point-blank boom!
                – Doktor J
                yesterday













              up vote
              7
              down vote










              up vote
              7
              down vote









              Much as I admire the sheer elegance of the sewage solution, I'd think the answer is obvious. Classic RPG tactics would suggest that if physical attacks don't work, use energy attacks. If your world is at industrial levels, use fire and lightning.



              Arrows and bags of manure, soaked in pitch and set alight before launch have been around since the Bronze Age. Even if the bags and contents disintegrate on impact, the fire won't and should severely burn large numbers of monsters if you use flaming bags.



              Steampunk era gives you both static and current electricity. Connect your walls to giant van de Graaff generators, then sit back and watch the monsters get electrocuted as they touch the conductive plates. Set up large capacitor arrays and use them to power arrow fired tasers to incinerate large targets.



              Since your monsters nullify, if not eat magic, you can't use mages, but mad science is always available to pick up the slack. If the above options work, upgrade to lasers, railguns and death rays.



              EDIT: For better results, incorporate @Alexis' solution. Put cheap magical items in your flaming loads and hopefully, the monsters will actually lump together to grab it.






              share|improve this answer














              Much as I admire the sheer elegance of the sewage solution, I'd think the answer is obvious. Classic RPG tactics would suggest that if physical attacks don't work, use energy attacks. If your world is at industrial levels, use fire and lightning.



              Arrows and bags of manure, soaked in pitch and set alight before launch have been around since the Bronze Age. Even if the bags and contents disintegrate on impact, the fire won't and should severely burn large numbers of monsters if you use flaming bags.



              Steampunk era gives you both static and current electricity. Connect your walls to giant van de Graaff generators, then sit back and watch the monsters get electrocuted as they touch the conductive plates. Set up large capacitor arrays and use them to power arrow fired tasers to incinerate large targets.



              Since your monsters nullify, if not eat magic, you can't use mages, but mad science is always available to pick up the slack. If the above options work, upgrade to lasers, railguns and death rays.



              EDIT: For better results, incorporate @Alexis' solution. Put cheap magical items in your flaming loads and hopefully, the monsters will actually lump together to grab it.







              share|improve this answer














              share|improve this answer



              share|improve this answer








              edited yesterday

























              answered 2 days ago









              nzaman

              8,68811443




              8,68811443








              • 1




                Additionally, if you lob out two-part liquid explosives (i.e. when both parts mix, boom) and then put low-effort runes on the material separating the components... when the payload lands, if the separator breaks on impact it still explodes damaging any nearby monsters; if it doesn't, then monsters are drawn to the runes and when they disintegrate the separator, you get point-blank boom!
                – Doktor J
                yesterday














              • 1




                Additionally, if you lob out two-part liquid explosives (i.e. when both parts mix, boom) and then put low-effort runes on the material separating the components... when the payload lands, if the separator breaks on impact it still explodes damaging any nearby monsters; if it doesn't, then monsters are drawn to the runes and when they disintegrate the separator, you get point-blank boom!
                – Doktor J
                yesterday








              1




              1




              Additionally, if you lob out two-part liquid explosives (i.e. when both parts mix, boom) and then put low-effort runes on the material separating the components... when the payload lands, if the separator breaks on impact it still explodes damaging any nearby monsters; if it doesn't, then monsters are drawn to the runes and when they disintegrate the separator, you get point-blank boom!
              – Doktor J
              yesterday




              Additionally, if you lob out two-part liquid explosives (i.e. when both parts mix, boom) and then put low-effort runes on the material separating the components... when the payload lands, if the separator breaks on impact it still explodes damaging any nearby monsters; if it doesn't, then monsters are drawn to the runes and when they disintegrate the separator, you get point-blank boom!
              – Doktor J
              yesterday










              up vote
              4
              down vote













              Build walls and weapons out of materials that cannot wither or crumble to ash. A fortress in the frozen North could be built out of ice. Pikes could be gold coated. Burning oil doesn't wither nor does boiling tar.



              In the end you need to destroy the portal letting them in. I'd build a fortress to hold them off and start a massive magical ritual for the purpose of attracting them as they seem to be creatures drive by instinct alone this way an attack force could attempt to close the portal from behind.






              share|improve this answer

























                up vote
                4
                down vote













                Build walls and weapons out of materials that cannot wither or crumble to ash. A fortress in the frozen North could be built out of ice. Pikes could be gold coated. Burning oil doesn't wither nor does boiling tar.



                In the end you need to destroy the portal letting them in. I'd build a fortress to hold them off and start a massive magical ritual for the purpose of attracting them as they seem to be creatures drive by instinct alone this way an attack force could attempt to close the portal from behind.






                share|improve this answer























                  up vote
                  4
                  down vote










                  up vote
                  4
                  down vote









                  Build walls and weapons out of materials that cannot wither or crumble to ash. A fortress in the frozen North could be built out of ice. Pikes could be gold coated. Burning oil doesn't wither nor does boiling tar.



                  In the end you need to destroy the portal letting them in. I'd build a fortress to hold them off and start a massive magical ritual for the purpose of attracting them as they seem to be creatures drive by instinct alone this way an attack force could attempt to close the portal from behind.






                  share|improve this answer












                  Build walls and weapons out of materials that cannot wither or crumble to ash. A fortress in the frozen North could be built out of ice. Pikes could be gold coated. Burning oil doesn't wither nor does boiling tar.



                  In the end you need to destroy the portal letting them in. I'd build a fortress to hold them off and start a massive magical ritual for the purpose of attracting them as they seem to be creatures drive by instinct alone this way an attack force could attempt to close the portal from behind.







                  share|improve this answer












                  share|improve this answer



                  share|improve this answer










                  answered 2 days ago









                  Thorne

                  13.5k42039




                  13.5k42039






















                      up vote
                      4
                      down vote













                      Do different materials crumble into different types of dust/ash? If so, perhaps the defenders can find a material whose dust/ash has an angle of respose is so steep that the monsters can't climb it.



                      Imagine trying to climb a mountain of powered sugar, you'd really just be wallowing in a pit of it before long, gradually eroding the mountain until the angle was such that you could actually approach.



                      The defenders would need to maintain their mountain of powder, as the energy imparted by the monsters would distribute it. I imagine a bucket-brigade of peasants in snow-shows, raking the powder back into the correct slope so that it is ready to ensnare the next batch of monsters.



                      Also, if the monsters are smart enough to build snow-shoes themselves, then this wont work.






                      share|improve this answer

























                        up vote
                        4
                        down vote













                        Do different materials crumble into different types of dust/ash? If so, perhaps the defenders can find a material whose dust/ash has an angle of respose is so steep that the monsters can't climb it.



                        Imagine trying to climb a mountain of powered sugar, you'd really just be wallowing in a pit of it before long, gradually eroding the mountain until the angle was such that you could actually approach.



                        The defenders would need to maintain their mountain of powder, as the energy imparted by the monsters would distribute it. I imagine a bucket-brigade of peasants in snow-shows, raking the powder back into the correct slope so that it is ready to ensnare the next batch of monsters.



                        Also, if the monsters are smart enough to build snow-shoes themselves, then this wont work.






                        share|improve this answer























                          up vote
                          4
                          down vote










                          up vote
                          4
                          down vote









                          Do different materials crumble into different types of dust/ash? If so, perhaps the defenders can find a material whose dust/ash has an angle of respose is so steep that the monsters can't climb it.



                          Imagine trying to climb a mountain of powered sugar, you'd really just be wallowing in a pit of it before long, gradually eroding the mountain until the angle was such that you could actually approach.



                          The defenders would need to maintain their mountain of powder, as the energy imparted by the monsters would distribute it. I imagine a bucket-brigade of peasants in snow-shows, raking the powder back into the correct slope so that it is ready to ensnare the next batch of monsters.



                          Also, if the monsters are smart enough to build snow-shoes themselves, then this wont work.






                          share|improve this answer












                          Do different materials crumble into different types of dust/ash? If so, perhaps the defenders can find a material whose dust/ash has an angle of respose is so steep that the monsters can't climb it.



                          Imagine trying to climb a mountain of powered sugar, you'd really just be wallowing in a pit of it before long, gradually eroding the mountain until the angle was such that you could actually approach.



                          The defenders would need to maintain their mountain of powder, as the energy imparted by the monsters would distribute it. I imagine a bucket-brigade of peasants in snow-shows, raking the powder back into the correct slope so that it is ready to ensnare the next batch of monsters.



                          Also, if the monsters are smart enough to build snow-shoes themselves, then this wont work.







                          share|improve this answer












                          share|improve this answer



                          share|improve this answer










                          answered 2 days ago









                          MatrixManAtYrService

                          1711




                          1711






















                              up vote
                              4
                              down vote













                              The monsters are smoke. The god of the void is the fire.



                              Your monsters are a symptom, like smoke is a symptom of fire. Smoke can definitely kill you and might be the worst thing about the fire, but it is difficult to fight smoke. You can avoid it or let it out or wear a mask but the smoke will keep coming, because the fire is still burning. To definitively fix a problem with smoke, you need to address the fire.



                              And here the fire is the God of the void. Ideas with how to come to terms with a divine entity whose interests are contrary to yours.



                              1: Come to terms. If the humans themselves facilitate the orderly destruction of magic items, that might be acceptable to the void god who would then be saved the trouble of making monsters. You would not want to destroy any faster than they can regenerate or this will not be sustainable/.



                              2: Distract void god into doing something else elsewhere. That worked for the good god, and so there are presumably elsewheres accessable to gods. Point out to the void god that it will meet less resistance and be able to destroy magic faster in a place where humans are not opposing it.



                              3: Recruit help. The existence of elsewheres and other gods suggest there might be other divine entities interested in preserving the status quo. One of these might be recruited to help humans oppose the void god as the good god used to - or maybe get that one back.



                              4: Best defense is a good offense. There are gods and there are gods. Maybe you can dissuade the god of the void from his goal somehow. Remember he is a god; it will take more than one shot.






                              share|improve this answer



























                                up vote
                                4
                                down vote













                                The monsters are smoke. The god of the void is the fire.



                                Your monsters are a symptom, like smoke is a symptom of fire. Smoke can definitely kill you and might be the worst thing about the fire, but it is difficult to fight smoke. You can avoid it or let it out or wear a mask but the smoke will keep coming, because the fire is still burning. To definitively fix a problem with smoke, you need to address the fire.



                                And here the fire is the God of the void. Ideas with how to come to terms with a divine entity whose interests are contrary to yours.



                                1: Come to terms. If the humans themselves facilitate the orderly destruction of magic items, that might be acceptable to the void god who would then be saved the trouble of making monsters. You would not want to destroy any faster than they can regenerate or this will not be sustainable/.



                                2: Distract void god into doing something else elsewhere. That worked for the good god, and so there are presumably elsewheres accessable to gods. Point out to the void god that it will meet less resistance and be able to destroy magic faster in a place where humans are not opposing it.



                                3: Recruit help. The existence of elsewheres and other gods suggest there might be other divine entities interested in preserving the status quo. One of these might be recruited to help humans oppose the void god as the good god used to - or maybe get that one back.



                                4: Best defense is a good offense. There are gods and there are gods. Maybe you can dissuade the god of the void from his goal somehow. Remember he is a god; it will take more than one shot.






                                share|improve this answer

























                                  up vote
                                  4
                                  down vote










                                  up vote
                                  4
                                  down vote









                                  The monsters are smoke. The god of the void is the fire.



                                  Your monsters are a symptom, like smoke is a symptom of fire. Smoke can definitely kill you and might be the worst thing about the fire, but it is difficult to fight smoke. You can avoid it or let it out or wear a mask but the smoke will keep coming, because the fire is still burning. To definitively fix a problem with smoke, you need to address the fire.



                                  And here the fire is the God of the void. Ideas with how to come to terms with a divine entity whose interests are contrary to yours.



                                  1: Come to terms. If the humans themselves facilitate the orderly destruction of magic items, that might be acceptable to the void god who would then be saved the trouble of making monsters. You would not want to destroy any faster than they can regenerate or this will not be sustainable/.



                                  2: Distract void god into doing something else elsewhere. That worked for the good god, and so there are presumably elsewheres accessable to gods. Point out to the void god that it will meet less resistance and be able to destroy magic faster in a place where humans are not opposing it.



                                  3: Recruit help. The existence of elsewheres and other gods suggest there might be other divine entities interested in preserving the status quo. One of these might be recruited to help humans oppose the void god as the good god used to - or maybe get that one back.



                                  4: Best defense is a good offense. There are gods and there are gods. Maybe you can dissuade the god of the void from his goal somehow. Remember he is a god; it will take more than one shot.






                                  share|improve this answer














                                  The monsters are smoke. The god of the void is the fire.



                                  Your monsters are a symptom, like smoke is a symptom of fire. Smoke can definitely kill you and might be the worst thing about the fire, but it is difficult to fight smoke. You can avoid it or let it out or wear a mask but the smoke will keep coming, because the fire is still burning. To definitively fix a problem with smoke, you need to address the fire.



                                  And here the fire is the God of the void. Ideas with how to come to terms with a divine entity whose interests are contrary to yours.



                                  1: Come to terms. If the humans themselves facilitate the orderly destruction of magic items, that might be acceptable to the void god who would then be saved the trouble of making monsters. You would not want to destroy any faster than they can regenerate or this will not be sustainable/.



                                  2: Distract void god into doing something else elsewhere. That worked for the good god, and so there are presumably elsewheres accessable to gods. Point out to the void god that it will meet less resistance and be able to destroy magic faster in a place where humans are not opposing it.



                                  3: Recruit help. The existence of elsewheres and other gods suggest there might be other divine entities interested in preserving the status quo. One of these might be recruited to help humans oppose the void god as the good god used to - or maybe get that one back.



                                  4: Best defense is a good offense. There are gods and there are gods. Maybe you can dissuade the god of the void from his goal somehow. Remember he is a god; it will take more than one shot.







                                  share|improve this answer














                                  share|improve this answer



                                  share|improve this answer








                                  edited yesterday

























                                  answered yesterday









                                  Willk

                                  95.5k25187405




                                  95.5k25187405






















                                      up vote
                                      3
                                      down vote













                                      Deep trenches and moats would be useful for slowing down the monsters without presenting them with matter that would be vulnerable to their disintegration ability. It would also help group them so they would be more vulnerable to artillery and massed fire.



                                      Depending on the exact nature of your magic system - a magical barrier such as a runic force shield may be much more resistant to attack than any wall made of a mundane material. Reinforcing your structures with magic to resist disintegration may also work.



                                      As for offence - melee is obviously inadvisable, as is allowing them to get to close to your defences really. Massed artillery fire seems to be the way to hold them at bay, bolstered by magic as much as possible. Depending on your technology level, weapons such as white phosphorous will allow you to level the playing field.



                                      Flamethrowers could also allow small units to effectively combat the monsters, though fuel usage may be a problem.



                                      Rune circles could be used a magical "honeypots" to lure creatures into a certain area before shelling the ever-loving beejeezus out of it from a safe distance. If magic can affect the monstyers, laying down area affect spells to weaken them as they run toward you defences may also be useful






                                      share|improve this answer

























                                        up vote
                                        3
                                        down vote













                                        Deep trenches and moats would be useful for slowing down the monsters without presenting them with matter that would be vulnerable to their disintegration ability. It would also help group them so they would be more vulnerable to artillery and massed fire.



                                        Depending on the exact nature of your magic system - a magical barrier such as a runic force shield may be much more resistant to attack than any wall made of a mundane material. Reinforcing your structures with magic to resist disintegration may also work.



                                        As for offence - melee is obviously inadvisable, as is allowing them to get to close to your defences really. Massed artillery fire seems to be the way to hold them at bay, bolstered by magic as much as possible. Depending on your technology level, weapons such as white phosphorous will allow you to level the playing field.



                                        Flamethrowers could also allow small units to effectively combat the monsters, though fuel usage may be a problem.



                                        Rune circles could be used a magical "honeypots" to lure creatures into a certain area before shelling the ever-loving beejeezus out of it from a safe distance. If magic can affect the monstyers, laying down area affect spells to weaken them as they run toward you defences may also be useful






                                        share|improve this answer























                                          up vote
                                          3
                                          down vote










                                          up vote
                                          3
                                          down vote









                                          Deep trenches and moats would be useful for slowing down the monsters without presenting them with matter that would be vulnerable to their disintegration ability. It would also help group them so they would be more vulnerable to artillery and massed fire.



                                          Depending on the exact nature of your magic system - a magical barrier such as a runic force shield may be much more resistant to attack than any wall made of a mundane material. Reinforcing your structures with magic to resist disintegration may also work.



                                          As for offence - melee is obviously inadvisable, as is allowing them to get to close to your defences really. Massed artillery fire seems to be the way to hold them at bay, bolstered by magic as much as possible. Depending on your technology level, weapons such as white phosphorous will allow you to level the playing field.



                                          Flamethrowers could also allow small units to effectively combat the monsters, though fuel usage may be a problem.



                                          Rune circles could be used a magical "honeypots" to lure creatures into a certain area before shelling the ever-loving beejeezus out of it from a safe distance. If magic can affect the monstyers, laying down area affect spells to weaken them as they run toward you defences may also be useful






                                          share|improve this answer












                                          Deep trenches and moats would be useful for slowing down the monsters without presenting them with matter that would be vulnerable to their disintegration ability. It would also help group them so they would be more vulnerable to artillery and massed fire.



                                          Depending on the exact nature of your magic system - a magical barrier such as a runic force shield may be much more resistant to attack than any wall made of a mundane material. Reinforcing your structures with magic to resist disintegration may also work.



                                          As for offence - melee is obviously inadvisable, as is allowing them to get to close to your defences really. Massed artillery fire seems to be the way to hold them at bay, bolstered by magic as much as possible. Depending on your technology level, weapons such as white phosphorous will allow you to level the playing field.



                                          Flamethrowers could also allow small units to effectively combat the monsters, though fuel usage may be a problem.



                                          Rune circles could be used a magical "honeypots" to lure creatures into a certain area before shelling the ever-loving beejeezus out of it from a safe distance. If magic can affect the monstyers, laying down area affect spells to weaken them as they run toward you defences may also be useful







                                          share|improve this answer












                                          share|improve this answer



                                          share|improve this answer










                                          answered 2 days ago









                                          Chromane

                                          2,837323




                                          2,837323






















                                              up vote
                                              3
                                              down vote













                                              Play Tower Defense



                                              If you present them with a solid wall they will attack it and eventually break through, so instead you create a funnel and maze setup as a killing ground.



                                              The walls have openings, that they will naturally funnel through. Inside the walls are a maze or labyrinth that forces them to run around a very long route.



                                              While they do that towers and other placements along the walls fire arrows down into them until they are weakened and die. The towers are linked by mobile wooden bridges.



                                              If they do turn and try to attack the walls or towers then burning oil is dumped on them. If they begin to climb the wall or it becomes too weak the defenders retreat to the next one and pull up the bridges, leaving the creatures stranded on an island and forced to climb down into the maze again.






                                              share|improve this answer





















                                              • This doesn't address the fliers though, who would presumably attack the emplacements from above. Otherwise a good strategy!
                                                – Doktor J
                                                yesterday















                                              up vote
                                              3
                                              down vote













                                              Play Tower Defense



                                              If you present them with a solid wall they will attack it and eventually break through, so instead you create a funnel and maze setup as a killing ground.



                                              The walls have openings, that they will naturally funnel through. Inside the walls are a maze or labyrinth that forces them to run around a very long route.



                                              While they do that towers and other placements along the walls fire arrows down into them until they are weakened and die. The towers are linked by mobile wooden bridges.



                                              If they do turn and try to attack the walls or towers then burning oil is dumped on them. If they begin to climb the wall or it becomes too weak the defenders retreat to the next one and pull up the bridges, leaving the creatures stranded on an island and forced to climb down into the maze again.






                                              share|improve this answer





















                                              • This doesn't address the fliers though, who would presumably attack the emplacements from above. Otherwise a good strategy!
                                                – Doktor J
                                                yesterday













                                              up vote
                                              3
                                              down vote










                                              up vote
                                              3
                                              down vote









                                              Play Tower Defense



                                              If you present them with a solid wall they will attack it and eventually break through, so instead you create a funnel and maze setup as a killing ground.



                                              The walls have openings, that they will naturally funnel through. Inside the walls are a maze or labyrinth that forces them to run around a very long route.



                                              While they do that towers and other placements along the walls fire arrows down into them until they are weakened and die. The towers are linked by mobile wooden bridges.



                                              If they do turn and try to attack the walls or towers then burning oil is dumped on them. If they begin to climb the wall or it becomes too weak the defenders retreat to the next one and pull up the bridges, leaving the creatures stranded on an island and forced to climb down into the maze again.






                                              share|improve this answer












                                              Play Tower Defense



                                              If you present them with a solid wall they will attack it and eventually break through, so instead you create a funnel and maze setup as a killing ground.



                                              The walls have openings, that they will naturally funnel through. Inside the walls are a maze or labyrinth that forces them to run around a very long route.



                                              While they do that towers and other placements along the walls fire arrows down into them until they are weakened and die. The towers are linked by mobile wooden bridges.



                                              If they do turn and try to attack the walls or towers then burning oil is dumped on them. If they begin to climb the wall or it becomes too weak the defenders retreat to the next one and pull up the bridges, leaving the creatures stranded on an island and forced to climb down into the maze again.







                                              share|improve this answer












                                              share|improve this answer



                                              share|improve this answer










                                              answered 2 days ago









                                              Tim B

                                              56.9k22160277




                                              56.9k22160277












                                              • This doesn't address the fliers though, who would presumably attack the emplacements from above. Otherwise a good strategy!
                                                – Doktor J
                                                yesterday


















                                              • This doesn't address the fliers though, who would presumably attack the emplacements from above. Otherwise a good strategy!
                                                – Doktor J
                                                yesterday
















                                              This doesn't address the fliers though, who would presumably attack the emplacements from above. Otherwise a good strategy!
                                              – Doktor J
                                              yesterday




                                              This doesn't address the fliers though, who would presumably attack the emplacements from above. Otherwise a good strategy!
                                              – Doktor J
                                              yesterday










                                              up vote
                                              3
                                              down vote













                                              Material of choice: Clay



                                              With earth being plentiful, it should be easy to rapidly produce clay weapons and barriers, using runes to reinforce them as much as needed.



                                              Weapons of Choice: Heavy Bludgeoning/Crushing class weapons.



                                              The nature of the void creatures means that penetration (piercing/slashing) class weapons will have more surface area exposed to the monsters, and thus will wither significantly faster. Plus the creatures seem to be susceptible to "bleeding" effects. Crushing class weapons like the war-hammer are capable of causing massive internal bleeding without ever penetrating the target. This makes Bludgeoning weapons the most potent and durable weapons to use.



                                              For flying enemies, The requirements of flight already require a light (frail) body. Their void ability making the air thin will require an extra light frame to remain airborne. Any projectile should easily rip these enemies to shreds. So standard archers with longbows should do. (You may be able to modify them to fire clay bolts)



                                              For elite enemies, projectiles can be enhanced with Greek fire or gunpowder (or a runic emulation of the effect) for some extra kick.



                                              Armor of Choice: Leather



                                              This should be plentiful enough, and allow the soldiers to take a few hits, but the lightness/flexibility should help them to avoid getting hit in the first place. Note that this means soldiers will get well rounded physical training. They need endurance+agility to avoid getting hit, but they also need strength to maximize the damage of their heavy weapons.



                                              Attack Scheduling



                                              Either using caravans of magically reinforced elite troops, or routinely creating/destroying runic fortifications, this should allow the frontier forts to cycle which are the most "magically fortified", allowing the strongest forts to fight hardest while the neighbor forts use the relative downtime to repair/fortify their defenses; as well as cycle active duty troops so that those on the front lines are always at the top of their game.



                                              This bundling of enemies will also make siege class weapons like catapults more effective as their projectiles can hit more enemies per volley.



                                              Side note: The psychological health of our troops is important to remember. We will want to rotate troops in and out of the front regularly. And while resting, it's important that they have plenty of emotional support.



                                              Fort Style: Rocky terrain



                                              The endless nature of this enemy makes traditional munition based defense of traditional forts unsustainable. We need to get the enemy close enough for our troops to crush them, while creating a heavily obstacle ridden environment to choke/slow the enemy to deny them any swarm advantage. Catapults behind this fortification (or deep in the middle) can use their munition store to wipe out large groups or particularly strong looking enemies before they reach the main line. The nature of this defense also means it will remain largely intact if we are forced to retreat. This means that our forts will be up and running the moment we take it back from the enemy.






                                              share|improve this answer

























                                                up vote
                                                3
                                                down vote













                                                Material of choice: Clay



                                                With earth being plentiful, it should be easy to rapidly produce clay weapons and barriers, using runes to reinforce them as much as needed.



                                                Weapons of Choice: Heavy Bludgeoning/Crushing class weapons.



                                                The nature of the void creatures means that penetration (piercing/slashing) class weapons will have more surface area exposed to the monsters, and thus will wither significantly faster. Plus the creatures seem to be susceptible to "bleeding" effects. Crushing class weapons like the war-hammer are capable of causing massive internal bleeding without ever penetrating the target. This makes Bludgeoning weapons the most potent and durable weapons to use.



                                                For flying enemies, The requirements of flight already require a light (frail) body. Their void ability making the air thin will require an extra light frame to remain airborne. Any projectile should easily rip these enemies to shreds. So standard archers with longbows should do. (You may be able to modify them to fire clay bolts)



                                                For elite enemies, projectiles can be enhanced with Greek fire or gunpowder (or a runic emulation of the effect) for some extra kick.



                                                Armor of Choice: Leather



                                                This should be plentiful enough, and allow the soldiers to take a few hits, but the lightness/flexibility should help them to avoid getting hit in the first place. Note that this means soldiers will get well rounded physical training. They need endurance+agility to avoid getting hit, but they also need strength to maximize the damage of their heavy weapons.



                                                Attack Scheduling



                                                Either using caravans of magically reinforced elite troops, or routinely creating/destroying runic fortifications, this should allow the frontier forts to cycle which are the most "magically fortified", allowing the strongest forts to fight hardest while the neighbor forts use the relative downtime to repair/fortify their defenses; as well as cycle active duty troops so that those on the front lines are always at the top of their game.



                                                This bundling of enemies will also make siege class weapons like catapults more effective as their projectiles can hit more enemies per volley.



                                                Side note: The psychological health of our troops is important to remember. We will want to rotate troops in and out of the front regularly. And while resting, it's important that they have plenty of emotional support.



                                                Fort Style: Rocky terrain



                                                The endless nature of this enemy makes traditional munition based defense of traditional forts unsustainable. We need to get the enemy close enough for our troops to crush them, while creating a heavily obstacle ridden environment to choke/slow the enemy to deny them any swarm advantage. Catapults behind this fortification (or deep in the middle) can use their munition store to wipe out large groups or particularly strong looking enemies before they reach the main line. The nature of this defense also means it will remain largely intact if we are forced to retreat. This means that our forts will be up and running the moment we take it back from the enemy.






                                                share|improve this answer























                                                  up vote
                                                  3
                                                  down vote










                                                  up vote
                                                  3
                                                  down vote









                                                  Material of choice: Clay



                                                  With earth being plentiful, it should be easy to rapidly produce clay weapons and barriers, using runes to reinforce them as much as needed.



                                                  Weapons of Choice: Heavy Bludgeoning/Crushing class weapons.



                                                  The nature of the void creatures means that penetration (piercing/slashing) class weapons will have more surface area exposed to the monsters, and thus will wither significantly faster. Plus the creatures seem to be susceptible to "bleeding" effects. Crushing class weapons like the war-hammer are capable of causing massive internal bleeding without ever penetrating the target. This makes Bludgeoning weapons the most potent and durable weapons to use.



                                                  For flying enemies, The requirements of flight already require a light (frail) body. Their void ability making the air thin will require an extra light frame to remain airborne. Any projectile should easily rip these enemies to shreds. So standard archers with longbows should do. (You may be able to modify them to fire clay bolts)



                                                  For elite enemies, projectiles can be enhanced with Greek fire or gunpowder (or a runic emulation of the effect) for some extra kick.



                                                  Armor of Choice: Leather



                                                  This should be plentiful enough, and allow the soldiers to take a few hits, but the lightness/flexibility should help them to avoid getting hit in the first place. Note that this means soldiers will get well rounded physical training. They need endurance+agility to avoid getting hit, but they also need strength to maximize the damage of their heavy weapons.



                                                  Attack Scheduling



                                                  Either using caravans of magically reinforced elite troops, or routinely creating/destroying runic fortifications, this should allow the frontier forts to cycle which are the most "magically fortified", allowing the strongest forts to fight hardest while the neighbor forts use the relative downtime to repair/fortify their defenses; as well as cycle active duty troops so that those on the front lines are always at the top of their game.



                                                  This bundling of enemies will also make siege class weapons like catapults more effective as their projectiles can hit more enemies per volley.



                                                  Side note: The psychological health of our troops is important to remember. We will want to rotate troops in and out of the front regularly. And while resting, it's important that they have plenty of emotional support.



                                                  Fort Style: Rocky terrain



                                                  The endless nature of this enemy makes traditional munition based defense of traditional forts unsustainable. We need to get the enemy close enough for our troops to crush them, while creating a heavily obstacle ridden environment to choke/slow the enemy to deny them any swarm advantage. Catapults behind this fortification (or deep in the middle) can use their munition store to wipe out large groups or particularly strong looking enemies before they reach the main line. The nature of this defense also means it will remain largely intact if we are forced to retreat. This means that our forts will be up and running the moment we take it back from the enemy.






                                                  share|improve this answer












                                                  Material of choice: Clay



                                                  With earth being plentiful, it should be easy to rapidly produce clay weapons and barriers, using runes to reinforce them as much as needed.



                                                  Weapons of Choice: Heavy Bludgeoning/Crushing class weapons.



                                                  The nature of the void creatures means that penetration (piercing/slashing) class weapons will have more surface area exposed to the monsters, and thus will wither significantly faster. Plus the creatures seem to be susceptible to "bleeding" effects. Crushing class weapons like the war-hammer are capable of causing massive internal bleeding without ever penetrating the target. This makes Bludgeoning weapons the most potent and durable weapons to use.



                                                  For flying enemies, The requirements of flight already require a light (frail) body. Their void ability making the air thin will require an extra light frame to remain airborne. Any projectile should easily rip these enemies to shreds. So standard archers with longbows should do. (You may be able to modify them to fire clay bolts)



                                                  For elite enemies, projectiles can be enhanced with Greek fire or gunpowder (or a runic emulation of the effect) for some extra kick.



                                                  Armor of Choice: Leather



                                                  This should be plentiful enough, and allow the soldiers to take a few hits, but the lightness/flexibility should help them to avoid getting hit in the first place. Note that this means soldiers will get well rounded physical training. They need endurance+agility to avoid getting hit, but they also need strength to maximize the damage of their heavy weapons.



                                                  Attack Scheduling



                                                  Either using caravans of magically reinforced elite troops, or routinely creating/destroying runic fortifications, this should allow the frontier forts to cycle which are the most "magically fortified", allowing the strongest forts to fight hardest while the neighbor forts use the relative downtime to repair/fortify their defenses; as well as cycle active duty troops so that those on the front lines are always at the top of their game.



                                                  This bundling of enemies will also make siege class weapons like catapults more effective as their projectiles can hit more enemies per volley.



                                                  Side note: The psychological health of our troops is important to remember. We will want to rotate troops in and out of the front regularly. And while resting, it's important that they have plenty of emotional support.



                                                  Fort Style: Rocky terrain



                                                  The endless nature of this enemy makes traditional munition based defense of traditional forts unsustainable. We need to get the enemy close enough for our troops to crush them, while creating a heavily obstacle ridden environment to choke/slow the enemy to deny them any swarm advantage. Catapults behind this fortification (or deep in the middle) can use their munition store to wipe out large groups or particularly strong looking enemies before they reach the main line. The nature of this defense also means it will remain largely intact if we are forced to retreat. This means that our forts will be up and running the moment we take it back from the enemy.







                                                  share|improve this answer












                                                  share|improve this answer



                                                  share|improve this answer










                                                  answered 2 days ago









                                                  Tezra

                                                  1,911734




                                                  1,911734






















                                                      up vote
                                                      2
                                                      down vote













                                                      Fire.



                                                      Nothing about these monsters seems to indicate that they can't burn.



                                                      The disintegration effect is less useful against things of lower density, so, petrol seems like a solid go-to delivery mechanism, but really what you'd want to go for is the highest possible temperature, so any good raging inferno should do.



                                                      You were less descriptive about the limitations of what runes can and can't do, but if they can produce fire or heat, it seems like making a reinforced runic pit of endless flame would be a pretty awesome defensive mechanism.






                                                      share|improve this answer





















                                                      • Oils (such as those rendered from animal fats or vegetables) have a double bonus -- not only are they of lower density, they're also organic in nature. They also burn quite well!
                                                        – Doktor J
                                                        yesterday















                                                      up vote
                                                      2
                                                      down vote













                                                      Fire.



                                                      Nothing about these monsters seems to indicate that they can't burn.



                                                      The disintegration effect is less useful against things of lower density, so, petrol seems like a solid go-to delivery mechanism, but really what you'd want to go for is the highest possible temperature, so any good raging inferno should do.



                                                      You were less descriptive about the limitations of what runes can and can't do, but if they can produce fire or heat, it seems like making a reinforced runic pit of endless flame would be a pretty awesome defensive mechanism.






                                                      share|improve this answer





















                                                      • Oils (such as those rendered from animal fats or vegetables) have a double bonus -- not only are they of lower density, they're also organic in nature. They also burn quite well!
                                                        – Doktor J
                                                        yesterday













                                                      up vote
                                                      2
                                                      down vote










                                                      up vote
                                                      2
                                                      down vote









                                                      Fire.



                                                      Nothing about these monsters seems to indicate that they can't burn.



                                                      The disintegration effect is less useful against things of lower density, so, petrol seems like a solid go-to delivery mechanism, but really what you'd want to go for is the highest possible temperature, so any good raging inferno should do.



                                                      You were less descriptive about the limitations of what runes can and can't do, but if they can produce fire or heat, it seems like making a reinforced runic pit of endless flame would be a pretty awesome defensive mechanism.






                                                      share|improve this answer












                                                      Fire.



                                                      Nothing about these monsters seems to indicate that they can't burn.



                                                      The disintegration effect is less useful against things of lower density, so, petrol seems like a solid go-to delivery mechanism, but really what you'd want to go for is the highest possible temperature, so any good raging inferno should do.



                                                      You were less descriptive about the limitations of what runes can and can't do, but if they can produce fire or heat, it seems like making a reinforced runic pit of endless flame would be a pretty awesome defensive mechanism.







                                                      share|improve this answer












                                                      share|improve this answer



                                                      share|improve this answer










                                                      answered yesterday









                                                      Iron Gremlin

                                                      5896




                                                      5896












                                                      • Oils (such as those rendered from animal fats or vegetables) have a double bonus -- not only are they of lower density, they're also organic in nature. They also burn quite well!
                                                        – Doktor J
                                                        yesterday


















                                                      • Oils (such as those rendered from animal fats or vegetables) have a double bonus -- not only are they of lower density, they're also organic in nature. They also burn quite well!
                                                        – Doktor J
                                                        yesterday
















                                                      Oils (such as those rendered from animal fats or vegetables) have a double bonus -- not only are they of lower density, they're also organic in nature. They also burn quite well!
                                                      – Doktor J
                                                      yesterday




                                                      Oils (such as those rendered from animal fats or vegetables) have a double bonus -- not only are they of lower density, they're also organic in nature. They also burn quite well!
                                                      – Doktor J
                                                      yesterday










                                                      up vote
                                                      1
                                                      down vote













                                                      Use their power against them



                                                      Section removed as OP updated they are immune to their own anti-material. See edit history for how that could have been done otherwise.



                                                      A more mundane approach



                                                      Ranged weapons, whether by length or by projectile, are what started to win wars later on anyway, and many melee weapons are lethal from just 1 or 2 hits. So really, although these enemies sound awful when described, they really aren't individually a lot worse than fighting other humans after all.



                                                      After re-reading the question again, I notice you suggest that arrows and guns just slow them down and it takes a lot of shots to kill them. In that case, arrows are less feasible in large scale attacks, but pole arms defending walls still work well. And when on the offensive, pole-arms behind trenches.



                                                      Also start using modern poaching techniques. Leave lots of hidden traps for them to step on so they come at your slower and weakened.



                                                      However, you did say there were huge numbers of them. That is probably the real problem here. But that is what castles are for. A castle allows its occupants to defend against a numerically vastly superior force, so again this just lends itself to what we see in our real history. However, if you rule that their touch can bring down walls too, then that might be problematic. However, some forts use earthwork fortifications, and disintegrating earth is not going to do much.



                                                      Summary



                                                      In a fantasy story, you can make their abilities sound awful, but in reality, against an army it isn't really all that bad and normal tactics which have been used in history can be used for the most part.



                                                      Since that is not so exciting, as the author you should find the exciting and dramatic middle ground and just rule that it works out however you want. The people don't really need anything special to overcome these enemies, as I stated, but you can just make the enemies seem powerful and go whatever direction you feel like.






                                                      share|improve this answer























                                                      • Ok some specifics on how their anti-material ability works(I'll edit this into the question as well). They are immune to it so you can't use them against each other. Its a full body effect including internals not just hands. They cannot be starved(its states in the question they need nothing to survive). And while an amusing mental image, capturing them and putting them on poles is so not viable, since they will simply disintegrate the ropes. They also disappear on death.
                                                        – ArcWraith
                                                        2 days ago















                                                      up vote
                                                      1
                                                      down vote













                                                      Use their power against them



                                                      Section removed as OP updated they are immune to their own anti-material. See edit history for how that could have been done otherwise.



                                                      A more mundane approach



                                                      Ranged weapons, whether by length or by projectile, are what started to win wars later on anyway, and many melee weapons are lethal from just 1 or 2 hits. So really, although these enemies sound awful when described, they really aren't individually a lot worse than fighting other humans after all.



                                                      After re-reading the question again, I notice you suggest that arrows and guns just slow them down and it takes a lot of shots to kill them. In that case, arrows are less feasible in large scale attacks, but pole arms defending walls still work well. And when on the offensive, pole-arms behind trenches.



                                                      Also start using modern poaching techniques. Leave lots of hidden traps for them to step on so they come at your slower and weakened.



                                                      However, you did say there were huge numbers of them. That is probably the real problem here. But that is what castles are for. A castle allows its occupants to defend against a numerically vastly superior force, so again this just lends itself to what we see in our real history. However, if you rule that their touch can bring down walls too, then that might be problematic. However, some forts use earthwork fortifications, and disintegrating earth is not going to do much.



                                                      Summary



                                                      In a fantasy story, you can make their abilities sound awful, but in reality, against an army it isn't really all that bad and normal tactics which have been used in history can be used for the most part.



                                                      Since that is not so exciting, as the author you should find the exciting and dramatic middle ground and just rule that it works out however you want. The people don't really need anything special to overcome these enemies, as I stated, but you can just make the enemies seem powerful and go whatever direction you feel like.






                                                      share|improve this answer























                                                      • Ok some specifics on how their anti-material ability works(I'll edit this into the question as well). They are immune to it so you can't use them against each other. Its a full body effect including internals not just hands. They cannot be starved(its states in the question they need nothing to survive). And while an amusing mental image, capturing them and putting them on poles is so not viable, since they will simply disintegrate the ropes. They also disappear on death.
                                                        – ArcWraith
                                                        2 days ago













                                                      up vote
                                                      1
                                                      down vote










                                                      up vote
                                                      1
                                                      down vote









                                                      Use their power against them



                                                      Section removed as OP updated they are immune to their own anti-material. See edit history for how that could have been done otherwise.



                                                      A more mundane approach



                                                      Ranged weapons, whether by length or by projectile, are what started to win wars later on anyway, and many melee weapons are lethal from just 1 or 2 hits. So really, although these enemies sound awful when described, they really aren't individually a lot worse than fighting other humans after all.



                                                      After re-reading the question again, I notice you suggest that arrows and guns just slow them down and it takes a lot of shots to kill them. In that case, arrows are less feasible in large scale attacks, but pole arms defending walls still work well. And when on the offensive, pole-arms behind trenches.



                                                      Also start using modern poaching techniques. Leave lots of hidden traps for them to step on so they come at your slower and weakened.



                                                      However, you did say there were huge numbers of them. That is probably the real problem here. But that is what castles are for. A castle allows its occupants to defend against a numerically vastly superior force, so again this just lends itself to what we see in our real history. However, if you rule that their touch can bring down walls too, then that might be problematic. However, some forts use earthwork fortifications, and disintegrating earth is not going to do much.



                                                      Summary



                                                      In a fantasy story, you can make their abilities sound awful, but in reality, against an army it isn't really all that bad and normal tactics which have been used in history can be used for the most part.



                                                      Since that is not so exciting, as the author you should find the exciting and dramatic middle ground and just rule that it works out however you want. The people don't really need anything special to overcome these enemies, as I stated, but you can just make the enemies seem powerful and go whatever direction you feel like.






                                                      share|improve this answer














                                                      Use their power against them



                                                      Section removed as OP updated they are immune to their own anti-material. See edit history for how that could have been done otherwise.



                                                      A more mundane approach



                                                      Ranged weapons, whether by length or by projectile, are what started to win wars later on anyway, and many melee weapons are lethal from just 1 or 2 hits. So really, although these enemies sound awful when described, they really aren't individually a lot worse than fighting other humans after all.



                                                      After re-reading the question again, I notice you suggest that arrows and guns just slow them down and it takes a lot of shots to kill them. In that case, arrows are less feasible in large scale attacks, but pole arms defending walls still work well. And when on the offensive, pole-arms behind trenches.



                                                      Also start using modern poaching techniques. Leave lots of hidden traps for them to step on so they come at your slower and weakened.



                                                      However, you did say there were huge numbers of them. That is probably the real problem here. But that is what castles are for. A castle allows its occupants to defend against a numerically vastly superior force, so again this just lends itself to what we see in our real history. However, if you rule that their touch can bring down walls too, then that might be problematic. However, some forts use earthwork fortifications, and disintegrating earth is not going to do much.



                                                      Summary



                                                      In a fantasy story, you can make their abilities sound awful, but in reality, against an army it isn't really all that bad and normal tactics which have been used in history can be used for the most part.



                                                      Since that is not so exciting, as the author you should find the exciting and dramatic middle ground and just rule that it works out however you want. The people don't really need anything special to overcome these enemies, as I stated, but you can just make the enemies seem powerful and go whatever direction you feel like.







                                                      share|improve this answer














                                                      share|improve this answer



                                                      share|improve this answer








                                                      edited 2 days ago

























                                                      answered 2 days ago









                                                      Aaron

                                                      2,381519




                                                      2,381519












                                                      • Ok some specifics on how their anti-material ability works(I'll edit this into the question as well). They are immune to it so you can't use them against each other. Its a full body effect including internals not just hands. They cannot be starved(its states in the question they need nothing to survive). And while an amusing mental image, capturing them and putting them on poles is so not viable, since they will simply disintegrate the ropes. They also disappear on death.
                                                        – ArcWraith
                                                        2 days ago


















                                                      • Ok some specifics on how their anti-material ability works(I'll edit this into the question as well). They are immune to it so you can't use them against each other. Its a full body effect including internals not just hands. They cannot be starved(its states in the question they need nothing to survive). And while an amusing mental image, capturing them and putting them on poles is so not viable, since they will simply disintegrate the ropes. They also disappear on death.
                                                        – ArcWraith
                                                        2 days ago
















                                                      Ok some specifics on how their anti-material ability works(I'll edit this into the question as well). They are immune to it so you can't use them against each other. Its a full body effect including internals not just hands. They cannot be starved(its states in the question they need nothing to survive). And while an amusing mental image, capturing them and putting them on poles is so not viable, since they will simply disintegrate the ropes. They also disappear on death.
                                                      – ArcWraith
                                                      2 days ago




                                                      Ok some specifics on how their anti-material ability works(I'll edit this into the question as well). They are immune to it so you can't use them against each other. Its a full body effect including internals not just hands. They cannot be starved(its states in the question they need nothing to survive). And while an amusing mental image, capturing them and putting them on poles is so not viable, since they will simply disintegrate the ropes. They also disappear on death.
                                                      – ArcWraith
                                                      2 days ago










                                                      up vote
                                                      1
                                                      down vote













                                                      So the best way for you to defend against the monsters forever is to take advantage of your naturally generating terrain. This means that your weapons should reply on regenerated materials, rather then complex processed materials. (I am approaching this from the same way I played a infamous game called Dwarf Fortress)



                                                      Defense 1 The Terrain



                                                      The biggest advantage you have will be the terrain. A close source of Lava or Water would be invaluable as it would allow you to harness this magical liquid to fight the beasts. The easiest way is to simply build a huge dam that holds all this liquid and release it into the oncoming horde every time they attack. This lets you simply wipe some of them away and slow their progress as they now have to walk through muddy ground while fighting the current from the burst dam. After the breach, you refill the dam using the natural regeneration of the terrain or allow the water/lava to slowly pour in. Depending on the source, you could have multiple dams or only use a limited amount to help delay the beasts while your other measures kick in.



                                                      You also want to limit where the monsters can approach from. Think of something like Helms deep from LOTR. Sure the monsters could approach from other directions, but they would never be able to do so in the numbers required to overtake your frontier forts (unless for some plot twist reason, you want to make it happen). Now the monsters might dig away at the sides and open up some passage ways, but when this happens you can send out some ranged forces to force them back and let it naturally regenerate over time.



                                                      Defense 2 Stone Catapults and Ballista



                                                      Terrain features regenerate overtime and there is always a abundance of stone to be found everywhere. Your siege weapons use this as a near infinite ammo. Not only will it regenerate over time providing you with a nearly unlimited source, its annoying for the monsters to remove. The battlie field will be littered with boulders and spears made out of stones which the monsters will have to push through. Combined with the a dam, you have a landslide that you can trigger once a battle. The monsters run in, you activate your dam. The boulders and spears from the previous battle are washed into the monsters and once the initial blast is over, you start firing, preparing the battlefield for the next battle.



                                                      Defense 3 Rune enforce everything



                                                      Runes stand out and attract more monsters. You want this, because you want to force your monsters to attack a fortress, not run around it. By providing a powerful enough incentive, you basically allowing the monsters to approach you the way you want them to. Reinforce all your defensive structures and watch the monsters pour in only to be washed out again. With the natural terrain supporting you, the monsters are going to find it extremely difficult to overcome you.






                                                      share|improve this answer





















                                                      • Rune enforcing everything is a good point to make. Focusing the monsters on the area most able to take them is definitely a good idea. As for the regenerating terrain, note the time it takes is highly dependent on scale. A simple rock on the side of the road could turn up in a month but a mountain would take centuries. Its also dependent on the position of the ley lines. An area without them would stay the same. The regeneration is also more dependant on natural processes. Erosion/chemical changes and the like would be influenced in the direction of ley with only minor matter creation
                                                        – ArcWraith
                                                        2 days ago












                                                      • Sir, I've come to inform you that trebuchets are the superior seige engine.
                                                        – tox123
                                                        yesterday










                                                      • @tox123 Even when trying to hit targets in the air?
                                                        – Shadowzee
                                                        yesterday










                                                      • @Shadowzee with enough time, effort, dedication, and practice, yes.
                                                        – tox123
                                                        yesterday















                                                      up vote
                                                      1
                                                      down vote













                                                      So the best way for you to defend against the monsters forever is to take advantage of your naturally generating terrain. This means that your weapons should reply on regenerated materials, rather then complex processed materials. (I am approaching this from the same way I played a infamous game called Dwarf Fortress)



                                                      Defense 1 The Terrain



                                                      The biggest advantage you have will be the terrain. A close source of Lava or Water would be invaluable as it would allow you to harness this magical liquid to fight the beasts. The easiest way is to simply build a huge dam that holds all this liquid and release it into the oncoming horde every time they attack. This lets you simply wipe some of them away and slow their progress as they now have to walk through muddy ground while fighting the current from the burst dam. After the breach, you refill the dam using the natural regeneration of the terrain or allow the water/lava to slowly pour in. Depending on the source, you could have multiple dams or only use a limited amount to help delay the beasts while your other measures kick in.



                                                      You also want to limit where the monsters can approach from. Think of something like Helms deep from LOTR. Sure the monsters could approach from other directions, but they would never be able to do so in the numbers required to overtake your frontier forts (unless for some plot twist reason, you want to make it happen). Now the monsters might dig away at the sides and open up some passage ways, but when this happens you can send out some ranged forces to force them back and let it naturally regenerate over time.



                                                      Defense 2 Stone Catapults and Ballista



                                                      Terrain features regenerate overtime and there is always a abundance of stone to be found everywhere. Your siege weapons use this as a near infinite ammo. Not only will it regenerate over time providing you with a nearly unlimited source, its annoying for the monsters to remove. The battlie field will be littered with boulders and spears made out of stones which the monsters will have to push through. Combined with the a dam, you have a landslide that you can trigger once a battle. The monsters run in, you activate your dam. The boulders and spears from the previous battle are washed into the monsters and once the initial blast is over, you start firing, preparing the battlefield for the next battle.



                                                      Defense 3 Rune enforce everything



                                                      Runes stand out and attract more monsters. You want this, because you want to force your monsters to attack a fortress, not run around it. By providing a powerful enough incentive, you basically allowing the monsters to approach you the way you want them to. Reinforce all your defensive structures and watch the monsters pour in only to be washed out again. With the natural terrain supporting you, the monsters are going to find it extremely difficult to overcome you.






                                                      share|improve this answer





















                                                      • Rune enforcing everything is a good point to make. Focusing the monsters on the area most able to take them is definitely a good idea. As for the regenerating terrain, note the time it takes is highly dependent on scale. A simple rock on the side of the road could turn up in a month but a mountain would take centuries. Its also dependent on the position of the ley lines. An area without them would stay the same. The regeneration is also more dependant on natural processes. Erosion/chemical changes and the like would be influenced in the direction of ley with only minor matter creation
                                                        – ArcWraith
                                                        2 days ago












                                                      • Sir, I've come to inform you that trebuchets are the superior seige engine.
                                                        – tox123
                                                        yesterday










                                                      • @tox123 Even when trying to hit targets in the air?
                                                        – Shadowzee
                                                        yesterday










                                                      • @Shadowzee with enough time, effort, dedication, and practice, yes.
                                                        – tox123
                                                        yesterday













                                                      up vote
                                                      1
                                                      down vote










                                                      up vote
                                                      1
                                                      down vote









                                                      So the best way for you to defend against the monsters forever is to take advantage of your naturally generating terrain. This means that your weapons should reply on regenerated materials, rather then complex processed materials. (I am approaching this from the same way I played a infamous game called Dwarf Fortress)



                                                      Defense 1 The Terrain



                                                      The biggest advantage you have will be the terrain. A close source of Lava or Water would be invaluable as it would allow you to harness this magical liquid to fight the beasts. The easiest way is to simply build a huge dam that holds all this liquid and release it into the oncoming horde every time they attack. This lets you simply wipe some of them away and slow their progress as they now have to walk through muddy ground while fighting the current from the burst dam. After the breach, you refill the dam using the natural regeneration of the terrain or allow the water/lava to slowly pour in. Depending on the source, you could have multiple dams or only use a limited amount to help delay the beasts while your other measures kick in.



                                                      You also want to limit where the monsters can approach from. Think of something like Helms deep from LOTR. Sure the monsters could approach from other directions, but they would never be able to do so in the numbers required to overtake your frontier forts (unless for some plot twist reason, you want to make it happen). Now the monsters might dig away at the sides and open up some passage ways, but when this happens you can send out some ranged forces to force them back and let it naturally regenerate over time.



                                                      Defense 2 Stone Catapults and Ballista



                                                      Terrain features regenerate overtime and there is always a abundance of stone to be found everywhere. Your siege weapons use this as a near infinite ammo. Not only will it regenerate over time providing you with a nearly unlimited source, its annoying for the monsters to remove. The battlie field will be littered with boulders and spears made out of stones which the monsters will have to push through. Combined with the a dam, you have a landslide that you can trigger once a battle. The monsters run in, you activate your dam. The boulders and spears from the previous battle are washed into the monsters and once the initial blast is over, you start firing, preparing the battlefield for the next battle.



                                                      Defense 3 Rune enforce everything



                                                      Runes stand out and attract more monsters. You want this, because you want to force your monsters to attack a fortress, not run around it. By providing a powerful enough incentive, you basically allowing the monsters to approach you the way you want them to. Reinforce all your defensive structures and watch the monsters pour in only to be washed out again. With the natural terrain supporting you, the monsters are going to find it extremely difficult to overcome you.






                                                      share|improve this answer












                                                      So the best way for you to defend against the monsters forever is to take advantage of your naturally generating terrain. This means that your weapons should reply on regenerated materials, rather then complex processed materials. (I am approaching this from the same way I played a infamous game called Dwarf Fortress)



                                                      Defense 1 The Terrain



                                                      The biggest advantage you have will be the terrain. A close source of Lava or Water would be invaluable as it would allow you to harness this magical liquid to fight the beasts. The easiest way is to simply build a huge dam that holds all this liquid and release it into the oncoming horde every time they attack. This lets you simply wipe some of them away and slow their progress as they now have to walk through muddy ground while fighting the current from the burst dam. After the breach, you refill the dam using the natural regeneration of the terrain or allow the water/lava to slowly pour in. Depending on the source, you could have multiple dams or only use a limited amount to help delay the beasts while your other measures kick in.



                                                      You also want to limit where the monsters can approach from. Think of something like Helms deep from LOTR. Sure the monsters could approach from other directions, but they would never be able to do so in the numbers required to overtake your frontier forts (unless for some plot twist reason, you want to make it happen). Now the monsters might dig away at the sides and open up some passage ways, but when this happens you can send out some ranged forces to force them back and let it naturally regenerate over time.



                                                      Defense 2 Stone Catapults and Ballista



                                                      Terrain features regenerate overtime and there is always a abundance of stone to be found everywhere. Your siege weapons use this as a near infinite ammo. Not only will it regenerate over time providing you with a nearly unlimited source, its annoying for the monsters to remove. The battlie field will be littered with boulders and spears made out of stones which the monsters will have to push through. Combined with the a dam, you have a landslide that you can trigger once a battle. The monsters run in, you activate your dam. The boulders and spears from the previous battle are washed into the monsters and once the initial blast is over, you start firing, preparing the battlefield for the next battle.



                                                      Defense 3 Rune enforce everything



                                                      Runes stand out and attract more monsters. You want this, because you want to force your monsters to attack a fortress, not run around it. By providing a powerful enough incentive, you basically allowing the monsters to approach you the way you want them to. Reinforce all your defensive structures and watch the monsters pour in only to be washed out again. With the natural terrain supporting you, the monsters are going to find it extremely difficult to overcome you.







                                                      share|improve this answer












                                                      share|improve this answer



                                                      share|improve this answer










                                                      answered 2 days ago









                                                      Shadowzee

                                                      5,829824




                                                      5,829824












                                                      • Rune enforcing everything is a good point to make. Focusing the monsters on the area most able to take them is definitely a good idea. As for the regenerating terrain, note the time it takes is highly dependent on scale. A simple rock on the side of the road could turn up in a month but a mountain would take centuries. Its also dependent on the position of the ley lines. An area without them would stay the same. The regeneration is also more dependant on natural processes. Erosion/chemical changes and the like would be influenced in the direction of ley with only minor matter creation
                                                        – ArcWraith
                                                        2 days ago












                                                      • Sir, I've come to inform you that trebuchets are the superior seige engine.
                                                        – tox123
                                                        yesterday










                                                      • @tox123 Even when trying to hit targets in the air?
                                                        – Shadowzee
                                                        yesterday










                                                      • @Shadowzee with enough time, effort, dedication, and practice, yes.
                                                        – tox123
                                                        yesterday


















                                                      • Rune enforcing everything is a good point to make. Focusing the monsters on the area most able to take them is definitely a good idea. As for the regenerating terrain, note the time it takes is highly dependent on scale. A simple rock on the side of the road could turn up in a month but a mountain would take centuries. Its also dependent on the position of the ley lines. An area without them would stay the same. The regeneration is also more dependant on natural processes. Erosion/chemical changes and the like would be influenced in the direction of ley with only minor matter creation
                                                        – ArcWraith
                                                        2 days ago












                                                      • Sir, I've come to inform you that trebuchets are the superior seige engine.
                                                        – tox123
                                                        yesterday










                                                      • @tox123 Even when trying to hit targets in the air?
                                                        – Shadowzee
                                                        yesterday










                                                      • @Shadowzee with enough time, effort, dedication, and practice, yes.
                                                        – tox123
                                                        yesterday
















                                                      Rune enforcing everything is a good point to make. Focusing the monsters on the area most able to take them is definitely a good idea. As for the regenerating terrain, note the time it takes is highly dependent on scale. A simple rock on the side of the road could turn up in a month but a mountain would take centuries. Its also dependent on the position of the ley lines. An area without them would stay the same. The regeneration is also more dependant on natural processes. Erosion/chemical changes and the like would be influenced in the direction of ley with only minor matter creation
                                                      – ArcWraith
                                                      2 days ago






                                                      Rune enforcing everything is a good point to make. Focusing the monsters on the area most able to take them is definitely a good idea. As for the regenerating terrain, note the time it takes is highly dependent on scale. A simple rock on the side of the road could turn up in a month but a mountain would take centuries. Its also dependent on the position of the ley lines. An area without them would stay the same. The regeneration is also more dependant on natural processes. Erosion/chemical changes and the like would be influenced in the direction of ley with only minor matter creation
                                                      – ArcWraith
                                                      2 days ago














                                                      Sir, I've come to inform you that trebuchets are the superior seige engine.
                                                      – tox123
                                                      yesterday




                                                      Sir, I've come to inform you that trebuchets are the superior seige engine.
                                                      – tox123
                                                      yesterday












                                                      @tox123 Even when trying to hit targets in the air?
                                                      – Shadowzee
                                                      yesterday




                                                      @tox123 Even when trying to hit targets in the air?
                                                      – Shadowzee
                                                      yesterday












                                                      @Shadowzee with enough time, effort, dedication, and practice, yes.
                                                      – tox123
                                                      yesterday




                                                      @Shadowzee with enough time, effort, dedication, and practice, yes.
                                                      – tox123
                                                      yesterday










                                                      up vote
                                                      1
                                                      down vote














                                                      Cutting them to pieces and blowing them up are effective at killing
                                                      them




                                                      If available fragmentation grenades.



                                                      If not whatever explosive you got combined with anything sharp. Nails, rocks with sharp edges, glass fragments.



                                                      Very simple to build and cuts them up badly and inflicts a lot of damage which is why terrorist use them.



                                                      You could add poison if it had any effect on them.






                                                      share|improve this answer

























                                                        up vote
                                                        1
                                                        down vote














                                                        Cutting them to pieces and blowing them up are effective at killing
                                                        them




                                                        If available fragmentation grenades.



                                                        If not whatever explosive you got combined with anything sharp. Nails, rocks with sharp edges, glass fragments.



                                                        Very simple to build and cuts them up badly and inflicts a lot of damage which is why terrorist use them.



                                                        You could add poison if it had any effect on them.






                                                        share|improve this answer























                                                          up vote
                                                          1
                                                          down vote










                                                          up vote
                                                          1
                                                          down vote










                                                          Cutting them to pieces and blowing them up are effective at killing
                                                          them




                                                          If available fragmentation grenades.



                                                          If not whatever explosive you got combined with anything sharp. Nails, rocks with sharp edges, glass fragments.



                                                          Very simple to build and cuts them up badly and inflicts a lot of damage which is why terrorist use them.



                                                          You could add poison if it had any effect on them.






                                                          share|improve this answer













                                                          Cutting them to pieces and blowing them up are effective at killing
                                                          them




                                                          If available fragmentation grenades.



                                                          If not whatever explosive you got combined with anything sharp. Nails, rocks with sharp edges, glass fragments.



                                                          Very simple to build and cuts them up badly and inflicts a lot of damage which is why terrorist use them.



                                                          You could add poison if it had any effect on them.







                                                          share|improve this answer












                                                          share|improve this answer



                                                          share|improve this answer










                                                          answered yesterday









                                                          cybernard

                                                          2,01836




                                                          2,01836






























                                                               

                                                              draft saved


                                                              draft discarded



















































                                                               


                                                              draft saved


                                                              draft discarded














                                                              StackExchange.ready(
                                                              function () {
                                                              StackExchange.openid.initPostLogin('.new-post-login', 'https%3a%2f%2fworldbuilding.stackexchange.com%2fquestions%2f129536%2fbest-strategy-against-anti-material-monsters%23new-answer', 'question_page');
                                                              }
                                                              );

                                                              Post as a guest




















































































                                                              Popular posts from this blog

                                                              鏡平學校

                                                              ꓛꓣだゔៀៅຸ໢ທຮ໕໒ ,ໂ'໥໓າ໼ឨឲ៵៭ៈゎゔit''䖳𥁄卿' ☨₤₨こゎもょの;ꜹꟚꞖꞵꟅꞛေၦေɯ,ɨɡ𛃵𛁹ޝ޳ޠ޾,ޤޒޯ޾𫝒𫠁သ𛅤チョ'サノބޘދ𛁐ᶿᶇᶀᶋᶠ㨑㽹⻮ꧬ꧹؍۩وَؠ㇕㇃㇪ ㇦㇋㇋ṜẰᵡᴠ 軌ᵕ搜۳ٰޗޮ޷ސޯ𫖾𫅀ल, ꙭ꙰ꚅꙁꚊꞻꝔ꟠Ꝭㄤﺟޱސꧨꧼ꧴ꧯꧽ꧲ꧯ'⽹⽭⾁⿞⼳⽋២៩ញណើꩯꩤ꩸ꩮᶻᶺᶧᶂ𫳲𫪭𬸄𫵰𬖩𬫣𬊉ၲ𛅬㕦䬺𫝌𫝼,,𫟖𫞽ហៅ஫㆔ాఆఅꙒꚞꙍ,Ꙟ꙱エ ,ポテ,フࢰࢯ𫟠𫞶 𫝤𫟠ﺕﹱﻜﻣ𪵕𪭸𪻆𪾩𫔷ġ,ŧآꞪ꟥,ꞔꝻ♚☹⛵𛀌ꬷꭞȄƁƪƬșƦǙǗdžƝǯǧⱦⱰꓕꓢႋ神 ဴ၀க௭எ௫ឫោ ' េㇷㇴㇼ神ㇸㇲㇽㇴㇼㇻㇸ'ㇸㇿㇸㇹㇰㆣꓚꓤ₡₧ ㄨㄟ㄂ㄖㄎ໗ツڒذ₶।ऩछएोञयूटक़कयँृी,冬'𛅢𛅥ㇱㇵㇶ𥄥𦒽𠣧𠊓𧢖𥞘𩔋цѰㄠſtʯʭɿʆʗʍʩɷɛ,əʏダヵㄐㄘR{gỚṖḺờṠṫảḙḭᴮᵏᴘᵀᵷᵕᴜᴏᵾq﮲ﲿﴽﭙ軌ﰬﶚﶧ﫲Ҝжюїкӈㇴffצּ﬘﭅﬈軌'ffistfflſtffतभफɳɰʊɲʎ𛁱𛁖𛁮𛀉 𛂯𛀞నఋŀŲ 𫟲𫠖𫞺ຆຆ ໹້໕໗ๆทԊꧢꧠ꧰ꓱ⿝⼑ŎḬẃẖỐẅ ,ờỰỈỗﮊDžȩꭏꭎꬻ꭮ꬿꭖꭥꭅ㇭神 ⾈ꓵꓑ⺄㄄ㄪㄙㄅㄇstA۵䞽ॶ𫞑𫝄㇉㇇゜軌𩜛𩳠Jﻺ‚Üမ႕ႌႊၐၸဓၞၞၡ៸wyvtᶎᶪᶹစဎ꣡꣰꣢꣤ٗ؋لㇳㇾㇻㇱ㆐㆔,,㆟Ⱶヤマފ޼ޝަݿݞݠݷݐ',ݘ,ݪݙݵ𬝉𬜁𫝨𫞘くせぉて¼óû×ó£…𛅑הㄙくԗԀ5606神45,神796'𪤻𫞧ꓐ㄁ㄘɥɺꓵꓲ3''7034׉ⱦⱠˆ“𫝋ȍ,ꩲ軌꩷ꩶꩧꩫఞ۔فڱێظペサ神ナᴦᵑ47 9238їﻂ䐊䔉㠸﬎ffiﬣ,לּᴷᴦᵛᵽ,ᴨᵤ ᵸᵥᴗᵈꚏꚉꚟ⻆rtǟƴ𬎎

                                                              Why https connections are so slow when debugging (stepping over) in Java?